osn fisika smp

77
Pembahasan Soal OSN Fisika SMP Tingkat Kabupaten/Kota Tahun 2010-2011 A. San Lohat Gurumuda.Net

Upload: jonni-maruli-tua-sitorus

Post on 09-Jul-2016

710 views

Category:

Documents


98 download

DESCRIPTION

soal soal osn

TRANSCRIPT

Page 1: osn fisika smp

Pembahasan Soal OSN Fisika SMPTingkat Kabupaten/Kota Tahun 2010-2011

A. San LohatGurumuda.Net

Page 2: osn fisika smp

Pembahasan Soal OSN Fisika SMP Tingkat Kabupaten/Kota Tahun 2010-2011

Daftar Isi

Daftar isi ................................................................................................................................................ iBesaran, satuan, dimensi, pengukuran dan angka penting ..................................................................... 1Resultan vektor, besaran vektor dan skalar ........................................................................................... 6Hukum Gerak Newton ........................................................................................................................... 8Gerak Lurus .......................................................................................................................................... 11Usaha dan Energi Mekanik, Pesawat Sederhana ................................................................................. 15Fluida Statis ......................................................................................................................................... 20Gelombang Bunyi ................................................................................................................................ 23Osilasi .................................................................................................................................................. 28Gelombang Mekanik ............................................................................................................................ 31Alat Optik Cermin dan Lensa .............................................................................................................. 35Alat Optik Mata, Kacamata dan Teropong .......................................................................................... 39Gelombang Cahaya .............................................................................................................................. 42Pemuaian .............................................................................................................................................. 47Skala Suhu, Kalibrasi termometer ....................................................................................................... 52Kalor, Perubahan Wujud, Perpindahan Kalor, Asas Black .................................................................. 55Muatan listrik, Gaya listrik .................................................................................................................. 60Rangkaian listrik, Hambatan listrik ..................................................................................................... 65Energi dan Daya listrik, Gerak Partikel Bermuatan ............................................................................ 68Hukum Ampere, GGL induksi, Transformator .................................................................................... 72

© 2015 Gurumuda.Net | A. San Lohat, S.Pd. i

Page 3: osn fisika smp

Pembahasan Soal OSN Fisika SMP Tingkat Kabupaten/Kota

Besaran, satuan, dimensi, pengukuran dan angka penting

Besaran dan satuan1. Soal Olimpiade Sains Nasional fisika SMP Tingkat Kabupaten/Kota Tahun 2010 No.1Perhatikan tabel hasil pengukuran empat orang siswa terhadap besaran fisika sebagai berikut!

Data dalam tabel tersebut yang benar dan berdasarkan satuan SI (Sistem Internasional) disajikan olehsiswa....A. R dan SB. P dan RC. P dan QD. Q dan SPembahasanSiswa P : Satuan Sistem Internasional (SI) besaran panjang adalah meter (m), bukan centimeter (cm)Alat ukur besaran panjang adalah mistarSiswa Q :Satuan Sistem Internasional (SI) besaran kuat arus listrik adalah Ampere (A)Alat ukur besaran kuat arus listrik adalah AmperemeterSiswa R :Satuan Sistem Internasional (SI) besaran suhu adalah Kelvin (K)Alat ukur besaran suhu adalah termometer, bukan hidrometer. Hidrometer adalah alat ukur massa jenis airSiswa S :Satuan Sistem Internasional (SI) besaran massa adalah Kilogram (Kg)Alat ukur besaran massa adalah neraca sama lengan

Data dalam tabel tersebut yang benar dan berdasarkan satuan SI (Sistem Internasional) disajikan olehsiswa Q dan S.Jawaban yang benar adalah D.

2. Soal Olimpiade Sains Nasional fisika SMP Tingkat Kabupaten/Kota Tahun 2011 No.2Hubungan besaran dan satuan yang benar adalah....

Besaran SatuanA Gaya Kg m s2

B Tekanan Kg m-2 s-2

C Daya Kg m-1 s-1

D Energi Kg m2 s-2

PembahasanGaya = Massa x PercepatanSatuan SI massa adalah kilogram (kg)Satuan SI percepatan adalah meter per sekon kuadrat (m/s2) Jadi satuan Gaya = kg m/s2 = kg m s-2

© 2015 Gurumuda.net | A. San Lohat, S.Pd. 1

Page 4: osn fisika smp

Pembahasan Soal OSN Fisika SMP Tingkat Kabupaten/Kota

Tekanan = Gaya / Luas PermukaanSatuan SI gaya adalah kg m/s2

Satuan SI luas permukaan adalah m2

Jadi satuan Tekanan = kg /m s2 = kg m-1 s-2

Energi = Usaha Usaha = Gaya x PerpindahanSatuan SI gaya adalah kilogram meter per sekon kuadrat (kg m/s2) Satuan SI perpindahan adalah meter (m)Jadi satuan Energi = satuan Usaha = kg m2/s2 = kg m2 s-2

Daya = Usaha / WaktuSatuan SI usaha adalah kilogram meter kuadrat per sekon kuadrat (kg m2/s2) Satuan SI waktu adalah sekon (s)Jadi satuan Daya = kg m2/s3 = kg m2 s-3

Jawaban yang benar adalah D.

Konversi satuan3. Soal Olimpiade Sains Nasional fisika SMP Tingkat Kabupaten/Kota Tahun 2010 No.2Setengah liter susu segar diperlukan sebagai bahan campuran saus puding, maka susu yang harusdimasukkan ke dalam bahan saus puding sebanyak....A. 5000 mlB. 500 ccC. 50 dm3

D. 5 m3

PembahasanDiketahui :Volume susu = 0,5 literDitanya : 0,5 liter = ..... ml = ..... cm3 (cc) = ..... dm3 = ..... m3

Jawab :1 liter = 1000 mililiter (1000 ml)1 liter = 0,001 m3 = 1 dm3 = 1000 cm3 (1000 cc)

0,5 liter = (0,5)(1000 ml) = 500 ml 0,5 dm3 = (0,5)(1000 cc) = 500 cc 0,5 liter = 0,5 dm3 = 0,0005 m3 Jawaban yang benar adalah B.

4. Soal Olimpiade Sains Nasional fisika SMP Tingkat Kabupaten/Kota Tahun 2011 No.1Jika ke dalam 100 mililiter susu cair ditambahkan 250 cc air maka dihasilkan air susu sebanyak....A. 12,5 dm3

B. 1,25 dm3

C. 3,5 dm3

D. 0,35 dm3

PembahasanDiketahui :Volume susu = 100 mlVolume air = 250 cc = 250 cm3

© 2015 Gurumuda.net | A. San Lohat, S.Pd. 2

Page 5: osn fisika smp

Pembahasan Soal OSN Fisika SMP Tingkat Kabupaten/Kota

Ditanya : 100 ml + 250 cc = ..... dm3

Jawab :ml = mililitercc = cm3 = sentimeter kubik dm3 = desimeter kubik

1 dm3 = 1 liter = 1000 mililiter sehingga 100 mililiter = 100/1000 liter = 1/10 liter = 0,1 liter = 0,1 dm3

1 dm3 = 1000 cm3 sehingga 250 cm3 = 250/1000 dm3 = 1/4 dm3 = 0,25 dm3

100 ml + 250 cm3 = 0,1 dm3 + 0,25 dm3 = 0,35 dm3 Jawaban yang benar adalah D.

5. Soal Olimpiade Sains Nasional fisika SMP Tingkat Kabupaten/Kota Tahun 2011 No.6Sebuah ruangan kelas yang panjangnya 10 m dan lebarnya 8 m, berlantai keramik yang berukuran 20cm x 20 cm. Jumlah keramik pada lantai dalam ruangan itu adalah....A. 20 buahB. 200 buahC. 2000 buahD. 20000 buahPembahasanDiketahui :Ukuran ruang kelas :Panjang = 10 meter Lebar = 8 meter Luas ruang kelas = 10 m x 8 m = 80 m2

Ukuran keramik :Panjang = 20 cmLebar = 20 cmLuas keramik = 20 cm x 20 cm = 400 cm2

Ditanya : Jumlah keramik pada lantai dalam ruanganJawab :1 m = 100 cm1 m2 = 10.000 cm2

Jadi 80 m2 = 80 x 10.000 cm2 = 800.000 cm2

Jumlah keramik = Luas ruang kelas / Luas keramikJumlah keramik = 800.000 cm2 / 400 cm2

Jumlah keramik = 2000 buahJawaban yang benar adalah C.

6. Soal Olimpiade Sains Nasional fisika SMP Tingkat Kabupaten/Kota Tahun 2011 No.9Setiap detik menetes 4 cc air dari akuarium yang bocor. Tetesan air ditampung di ember berukuran 5liter. Ember terisi penuh dalam waktu....A. 125 detikB. 200 detikC. 1250 detik

© 2015 Gurumuda.net | A. San Lohat, S.Pd. 3

Page 6: osn fisika smp

Pembahasan Soal OSN Fisika SMP Tingkat Kabupaten/Kota

D. 2000 detikPembahasanDiketahui :1 liter = 1 dm3 = 1000 cm3

Volume air = 4 cc = 4 cm3

Volume ember = 5 liter = 5 dm3 = 5 x 1000 cm3 = 5000 cm3

Ditanya : Selang waktu ember terisi penuh Jawab :5000 cm3 / 4 cm3 = 1250 detikJawaban yang benar adalah C.

Awalan satuan7. Soal Olimpiade Sains Nasional fisika SMP Tingkat Kabupaten/Kota Tahun 2011 No.3Urutan awalan untuk satuan dari besar ke kecil yang benar adalah....A. Mega, terra, nano, pikoB. Giga, Mega, Mikro, nanoC. Piko, Nano, Mega, TerraD. Mega, Terra, Mikro, PikoPembahasanAwalan

Singkatan

Kelipatan

Nilai

Terra T 1012 1.000.000.000.000

Giga G 109 1.000.000.000Mega M 106 1.000.000Kilo K 103 1000

Mili m 10-3 0,001Mikro µ 10-6 0,000001Nano n 10-9 0,000000001Piko p 10--12 0,00000000000

1Jawaban yang benar adalah B.

Analisis dimensi8. Soal Olimpiade Sains Nasional SMP Tingkat Kabupaten/Kota Tahun 2011 No.5Berdasarkan analisis dimensi, besaran-besaran fisika di bawah ini yang berdimensi sama adalah...A. usaha dan dayaB. daya dan tekananC. gaya dan momentumD. usaha dan kalorPembahasanDiketahui :Dimensi Dimensi Panjang = Dimensi Lebar = Dimensi Perpindahan = LDimensi Waktu = TDimensi Massa = MJawab :Dimensi Luas = Dimensi Panjang x Dimensi Lebar = [L][L] = [L]2

© 2015 Gurumuda.net | A. San Lohat, S.Pd. 4

Page 7: osn fisika smp

Pembahasan Soal OSN Fisika SMP Tingkat Kabupaten/Kota

Dimensi Kecepatan = Dimensi Perpindahan / Dimensi Waktu = [L] / [T] = [L][T]-1

Dimensi Percepatan = Dimensi Kecepatan / Dimensi Waktu = [L][T]-1 / [T] = [L][T]-1[T]-1 = [L][T]-2 Dimensi Gaya = Dimensi Massa x Dimensi Percepatan = [M][L][T]-2 Dimensi Usaha = Dimensi Gaya x Dimensi Perpindahan = [M][L][T]-2 [L] = [M][L]2[T]-2

Dimensi Daya = Dimensi Usaha / Dimensi Waktu = [M][L]2[T]-2 / [T] = [M][L]2[T]-2[T]-1 = [M][L]2[T]-

3 Dimensi Tekanan = Dimensi Gaya / Dimensi Luas = [M][L][T]-2 / [L]2 = [M][L][T]-2 [L]-2 = [M] [L]-1[T]-

2 Dimensi Momentum = Dimensi Massa x Dimensi Kecepatan = [M][L][T]-1

Dimensi Kalor = Dimensi Energi = Dimensi Usaha = [M][L]2[T]-2 Jawaban yang benar adalah D.

Catatan :Kalor adalah energi yang berpindah karena perbedaan suhu. Usaha adalah proses perpindahan energi melalui perpindahan benda. Jadi dimensi kalor = dimensi energi = dimensi usaha.

Pengukuran dan angka penting9. Soal Olimpiade Sains Nasional fisika SMP Tingkat Kabupaten/Kota Tahun 2011 No.7Panjang sebuah benda diukur dengan menggunakan mikrometer dan terbaca nilai sebesar 15,08 mm.Bilangan penting dan jumlah angka penting hasil pengukuran itu berturut-turut adalah ....A. 15,08 mm dan 3 angka pentingB. 1,508 mm dan 4 angka pentingC. 15,08 mm dan 4 angka pentingD. 1,508 mm dan 3 angka pentingPembahasanBilangan penting = 15,08 milimeterJumlah angka penting = 4Jawaban yang benar adalah C.

© 2015 Gurumuda.net | A. San Lohat, S.Pd. 5

Page 8: osn fisika smp

Pembahasan Soal OSN Fisika SMP Tingkat Kabupaten/Kota

Resultan vektor, besaran vektor dan skalar

Resultan Dua Vektor Gaya1. Soal Olimpiade Sains Nasional fisika SMP Tingkat Kabupaten/Kota Tahun 2010 No.3Dua orang anak A dan B mendorong balok, jika A mendorong balok ke selatan dengan gaya 400 N danB saat yang sama mendorong balok ke arah timur dengan gaya 300 N, maka resultan gaya A dan B ....A. 100 N ke selatanB. 100 N ke timurC. 500 N ke tenggaraD. 700 N ke tenggaraPembahasanDiketahui :

Keterangan gambar :U = utara, T = timur, S = selatan, B = baratTL = timur laut, TG = tenggara, BD = barat daya, BL = barat laut

A = 400 Newton ke selatanB = 300 Newtonn ke timurDitanya : besar dan arah resultan gaya (R)Jawab :

R=√ A2+B2

=√4002+3002

R=√160.000+90.000R=√250.000R=500 Newton, ke tenggara

Jawaban yang benar adalah C.

Resultan Vektor Perpindahan2. Soal Olimpiade Sains Nasional fisika SMP Tingkat Kabupaten/Kota Tahun 2011 No.4Seseorang memacu sepeda motor dari rumahnya ke arah utara sejauh 6 km, lalu berbelok ke timursejauh 8 km. Posisi orang tersebut dari rumahnya setelah melakukan perjalanan tersebut adalah....A. 14 km ke arah timur lautB. 14 km ke arah barat dayaC. 10 km ke arah timur lautD. 10 km ke arah barat lautPembahasanDiketahui :

Ditanya : besar dan arah resultan perpindahan Jawab :

R=√62+82=√36+64

R=√100=10 km, ke timur lautJawaban yang benar adalah C.

© 2015 Gurumuda.net | A. San Lohat, S.Pd. 6

Page 9: osn fisika smp

Pembahasan Soal OSN Fisika SMP Tingkat Kabupaten/Kota

Besaran Vektor dan Skalar3. Soal Olimpiade Sains Nasional fisika SMP Tingkat Kabupaten/Kota Tahun 2011 No.8Di antara pilihan berikut, yang merupakan pasangan besaran skalar – vektor adalah....A. Gaya - percepatanB. Tekanan - gayaC. Perpindahan - lajuD. Kuat arus listrik - tekananPembahasanGaya = besaran vektor - percepatan = besaran vektorTekanan = besaran skalar – gaya = besaran vektorPerpindahan = besaran vektor – laju = besaran skalarKuat arus listrik = besaran skalar – tekanan = besaran skalarJawaban yang benar adalah B.

© 2015 Gurumuda.net | A. San Lohat, S.Pd. 7

Page 10: osn fisika smp

Pembahasan Soal OSN Fisika SMP Tingkat Kabupaten/Kota Gurumuda.Net

Hukum Gerak Newton

Hukum I Newton1. Soal Olimpiade Sains Nasional fisika SMP Tingkat Kabupaten/Kota Tahun 2010 No.8Seseorang berada di dalam sebuah lift yang sedang bergerak dengan kecepatan konstan ke atas. beratorang itu 800 N. Tiba-tiba tali baja penarik lift putus, sehingga lift jatuh. Gaya normal dari lantai liftpada orang itu sesaat sebelum dan sesudah tali baja putus adalah....A. 800 N dan 0B. 800 N dan 800 NC. 1600 N dan 0D. 1600 N dan 800 NPembahasanDiketahui :Berat (w) = 800 NewtonDitanya : Gaya normal (N) yang bekerja pada orang itu sesaat sebelum dan sesudah tali baja putusJawab :Sesaat sebelum tali baja putusKetika orang itu berdiri di atas lantai lift, gaya berat bekerja pada orang itu dan arah gaya berat kebawah (amati gambar ilustrasi di bawah). Orang itu diam dalam lift sehingga harus ada gaya normalyang mengimbangi gaya berat orang itu, di mana gaya ini mempunyai arah ke atas dan besarnya samadengan gaya berat. Gaya normal dikerjakan lantai pada orang itu dan gaya ini bekerja pada orang.

Karena orang itu diam dalam lift dan lift bergerak dengan kecepatankonstan (tidak ada percepatan) maka resultan gaya yang bekerja padaorang bernilai nol.∑F = 0N – w = 0N = w

N = 800 Newton

Setelah tali baja putusSetelah tali baja putus, lift dan orang itu jatuh bebas secara bersama-sama, di mana besar dan arahpercepatan jatuh bebas lift dan orang itu bernilai sama dengan percepatan gravitasi bumi. Lift danorang itu jatuh bebas secara bersama-sama sehingga tidak ada gaya normal yang dikerjakan lantai liftpada orang itu. Orang itu sedang jatuh dan kaki orang itu juga tidak bisa menekan lantai lift karena liftjuga sedang jatuh. Jawaban yang benar adalah A.

Hukum II Newton2. Soal Olimpiade Sains Nasional SMP Tingkat Kabupaten/Kota Tahun 2010 No.4Perhatikan gambar!

Sebuah balok 40 kg ditarik dengan gaya 200 N timbul percepatan 3 m/s2 maka besar gaya gesek yangterjadi pada benda/balok tersebut adalah....A. 15 NB. 40 N

© 2015 Gurumuda.net | A. San Lohat, S.Pd. 8

Page 11: osn fisika smp

Pembahasan Soal OSN Fisika SMP Tingkat Kabupaten/Kota Gurumuda.Net

C. 43 ND. 80 NPembahasanDiketahui :Massa (m) = 40 kgGaya (F) = 200 NPercepatan (a) = 3 m/s2

Ditanya : Besar gaya gesek (Fg)Jawab :Rumus hukum II Newton :∑F = m aKeterangan : ∑F = resultan gaya, m = massa, a = percepatan

Arah gaya F ke kanan sedangkan arah gaya gesek ke kiri (arah gaya gesek berlawanan dengan arahgerak benda). Arah ke kanan dipilih positif dan arah ke kiri dipilih negatif.∑F = m aF – Fg = m a200 – Fg = (40)(3)200 – Fg = 120Fg = 200 – 120Fg = 80 NewtonJawaban yang benar adalah D.

3. Soal Olimpiade Sains Nasional SMP Tingkat Kabupaten/Kota Tahun 2011 No.11Balok A yang massanya 100 gram diletakkan di atas balok B yang massanya 300 gram, kemudianbalok B didorong oleh gaya 5 N vertikal ke atas. Jika balok tidak saling bergerak, maka besar gayanormal oleh balok B pada balok A adalah....A. 1 NB. 1,25 NC. 2 ND. 3 NPembahasanDiketahui :Gaya dorong (F) = 5 Newton

Massa balok A (mA) = 100 gram = 0,1 kg Massa balok B (mB) = 300 gram = 0,3 kgPercepatan gravitasi (g) = 10 m/s2

Berat balok A (wA) = (0,1 kg)(10 m/s2) = 1 kg m/s2 = 1 NewtonBerat balok B (wB) = (0,3 kg)(10 m/s2) = 3 kg m/s2 = 3 Newton

Ditanya : gaya Normal oleh balok B pada balok AJawab :Terdapat beberapa gaya yang bekerja pada kedua balok, sebagaimana ditunjukkan pada gambar dibawah.

© 2015 Gurumuda.net | A. San Lohat, S.Pd. 9

Page 12: osn fisika smp

Pembahasan Soal OSN Fisika SMP Tingkat Kabupaten/Kota Gurumuda.Net

F = gaya dorong (bekerja pada balok B)wA = gaya berat balok A (bekerja pada balok A)wB = gaya berat balok B (bekerja pada balok B)NA = gaya normal oleh balok B pada balok A (bekerja pada balok A)NA’ = gaya normal oleh balok A pada balok B (bekerja pada balok B)

Tinjau kedua balok sebagai satu sistem :Terapkan hukum II Newton pada kedua balok :∑F = m a

F – wA – wB + NA – NA’ = (mA + mB) aNA dan NA’ merupakan gaya aksi-reaksi yang mempunyai besar sama tapi berlawanan arah sehinggadilenyapkan dari persamaan.F – wA – wB = (mA + mB) a5 – 1 – 3 = (0,1 + 0,3) a5 – 4 = (0,4) a1 = (0,4) aa = 1 / 0,4a = 2,5 m/s2

Adanya gaya (F) 5 Newton bekerja vertikal ke atas pada kedua balok menyebabkan kedua balokbergerak vertikal ke atas dengan percepatan sebesar 2,5 m/s2.

Tinjau balok A :Terapkan hukum II Newton pada balok A :∑F = m aNA – wA = mA aNA – 1 = (0,1)(2,5)NA – 1 = 0,25NA = 1 + 0,25NA = 1,25 NewtonGaya Normal oleh balok B pada balok A adalah 1,25 Newton.Jawaban yang benar adalah B.

© 2015 Gurumuda.net | A. San Lohat, S.Pd. 10

Page 13: osn fisika smp

Pembahasan Soal OSN Fisika SMP Tingkat Kabupaten/Kota

Gerak Lurus

Gerak Jatuh Bebas1. Soal Olimpiade Sains Nasional fisika SMP Tingkat Kabupaten/Kota Tahun 2010 No.5Perhatikan gambar berikut!

Gerakan kelapa jatuh (gambar I) dan gerak bola yang dilempar vertikal ke atas hingga titik tertinggioleh seorang siswa (gambar II) memiliki jenis gerak....

PembahasanGambar 1 = gerak jatuh bebas = gerak lurus berubah beraturan (GLBB) dipercepatGambar 2 = gerak vertikal ke atas = gerak lurus berubah beraturan (GLBB) diperlambatJawaban yang benar adalah A.

Grafik Gerak Lurus2. Soal Olimpiade Sains Nasional fisika SMP Tingkat Kabupaten/Kota Tahun 2011 No.12Jika benda dilempar ke atas dari permukaan bumi, maka grafik besar percepatan yang dialami bendaadalah....

PembahasanKetika benda dilempar ke atas dari permukaan bumi, percepatan yang dialami benda adalah percepatangravitasi bumi. Besar percepatan gravitasi bumi bernilai konstan yakni 9,8 m/s2 dan arah percepatangravitasi bumi selalu menuju pusat bumi.Percepatan konstan ditandai oleh garis lurus sejajar sumbu t dan tegak lurus dengan sumbu a.Jawaban yang benar adalah D.

3. Soal Olimpiade Sains Nasional fisika SMP Tingkat Kabupaten/Kota Tahun 2011 No.13

© 2015 Gurumuda.net | A. San Lohat, S.Pd. 11

Page 14: osn fisika smp

Pembahasan Soal OSN Fisika SMP Tingkat Kabupaten/Kota

Sebuah benda bergerak dengan laju yang digambarkan sepertipada grafik di samping ini. Jarak yang ditempuh benda selama 20detik adalah....A. 600 mB. 500 mC. 200 mD. 100 mPembahasanJarak tempuh selama 0 - 10 sekon = luas persegi + luas segitigaLuas persegi = (20-0)(10-0) = (20)(10) = 200 meterLuas segitiga = (1/2)(10-0)(40-20) = (1/2)(10)(20) = (5)(20) = 100 meterJarak tempuh selama 0 – 10 sekon = 200 meter + 100 meter = 300 meter

Jarak tempuh selama 10 – 20 sekon = luas segitigaLuas segitiga = (1/2)(20-10)(40-0) = (1/2)(10)(40) = (5)(40) = 200 meter

Jarak tempuh benda selama 0 - 20 sekon :300 meter + 200 meter = 500 meter

Jawaban yang benar adalah B.

4. Soal Olimpiade Sains Nasional fisika SMP Tingkat Kabupaten/Kota Tahun 2011 No.14Gerak tiga buah benda, masing-masing digambarkan dengan grafik di bawah ini.

Pernyataan yang benar untuk gerak ketiga benda adalah....

PembahasanGrafik benda 1 = Grafik percepatan (a) dan waktu (t)Berdasarkan grafik, benda bergerak dengan percepatan konstan atau benda bergerak lurus berubahberaturan. Percepatan konstan ditunjukkan oleh garis lurus yang memotong sumbu percepatan (a).

Grafik benda 2 = Grafik kecepatan (v) dan waktu (a)Berdasarkan grafik, benda bergerak dengan kecepatan konstan atau benda bergerak lurus beraturan.Kecepatan konstan ditunjukkan oleh garis lurus yang memotong sumbu kecepatan (v).

Grafik benda 3 = Grafik perpindahan (x) dan waktu (a)

© 2015 Gurumuda.net | A. San Lohat, S.Pd. 12

Page 15: osn fisika smp

Pembahasan Soal OSN Fisika SMP Tingkat Kabupaten/Kota

Berdasarkan grafik, tampak perpindahan benda selalu tetap. Ini artinya benda tidak bergerak atau bendadiam. Benda diam ditunjukkan oleh garis lurus yang memotong sumbu perpindahan (x).Jawaban yang benar adalah A.

5. Soal Olimpiade Sains Nasional fisika SMP Tingkat Kabupaten/Kota Tahun 2011 No.19Posisi sebuah benda yang bergerak di sepanjang sumbu x ditunjukkan oleh grafik berikut ini.

Grafik tersebut menunjukkan bahwa benda bergerak dengan laju konstan pada selang waktu antara....A. 5 s/d 15 detik dan 25 s/d 35 detikB. 0 s/d 5 detik dan 35 s/d 40 detikC. 15 s/d 25 detikD. 0 s/d 5 detik, 15 s/d 25 detik dan 35 s/d 40 detikPembahasan5 s/d 15 detik dan 25 s/d 35 detik = jarak benda selalu tetap atau benda diam0 s/d 5 detik, 15 s/d 25 detik dan 35 s/d 40 detik = jarak benda selalu berubah secara teratur selangwaktu yang sama atau benda bergerak dengan laju konstan.Jawaban yang benar adalah D.

Kecepatan6. Soal Olimpiade Sains Nasional fisika SMP Tingkat Kabupaten/Kota Tahun 2010 No.7Seorang pemburu dengan mobilnya sedang mengejar seekor rusa. Mobil melaju dengan kecepatan 72km/jam dan rusa berlari dengan kecepatan 64,8 km/jam. Ketika jarak antara mobil dan rusa 2012 msang pemburu menembakkan senapannya. Peluru keluar dari laras senapan dengan laju 200 m/sterhadap laras senapan. Peluru mengenai rusa setelah waktu....A. 0,5 sekonB. 1 sekonC. 1,25 sekonD. 1,5 sekonPembahasanDiketahui :Kelajuan mobil (vb) = 72 km/jam = (72)(1000 m) / 3600 s = 20 m/s Kelajuan rusa (vr) = 64,8 km/jam = (64,8)(1000 m) / 3600 s = 64800 m / 3600 s = 18 m/s Ketika senapan ditembakkan, jarak antara mobil dan rusa (s) = 202 meterKelajuan peluru (vp) = 20 m/s + 200 m/s = 220 m/sSenjata dipegang pemburu yang berada di dalam mobil yang bergerak dengan kelajuan 20 m/ssehingga kelajuan peluru ditambahkan juga dengan kelajuan mobil. Ditanya : Peluru mengenai rusa setelah selang waktu....Jawab :Anggap saja mobil dan rusa bergerak lurus dengan kecepatan tetap.Rumus kecepatan : v = s / t atau s = v tKeterangan rumus : v = kelajuan, s = jarak, t = waktu

© 2015 Gurumuda.net | A. San Lohat, S.Pd. 13

Page 16: osn fisika smp

Pembahasan Soal OSN Fisika SMP Tingkat Kabupaten/Kota

Jarak tempuh rusa = 202 + Xr = 202 + vr t = 202 + 18 tJarak tempuh peluru = Yp = vp t = 220 t

Ketika peluru mengenai rusa, jarak tempuh peluru = jarak tempuh rusa. Jarak tempuh rusa = Jarak tempuh peluru202 + 18 t = 220 t202 = 220 t – 18 t202 = 202 tt = 202/202 t = 1 sekonJawaban yang benar adalah B.

© 2015 Gurumuda.net | A. San Lohat, S.Pd. 14

Page 17: osn fisika smp

Pembahasan Soal OSN Fisika SMP Tingkat Kabupaten/Kota Gurumuda.Net

Usaha, teorema usaha-energi kinetik, hukum kekekalan energi mekanik, energi potensialpegas, pesawat sederhana

Usaha1. Soal Olimpiade Sains Nasional fisika SMP Tingkat Kabupaten/Kota Tahun 2010 No.9Seorang cucu mendorong kursi roda neneknya di sebuah taman yang berbentuk lingkaran dengan gaya20 N, jari-jari taman tersebut 7 m, berapa besar usaha yang dilakukan si cucu untuk 2 kali putaran ?A. 0 JouleB. 1400 JouleC. 1540 JouleD. 1760 JoulePembahasanRumus Usaha : Usaha = Gaya x PerpindahanJika si cucu mendorong kursi roda sejauh 2 kali putaran maka si cucu dan kursi roda kembali ke posisisemula. Karena kembali ke posisi semula maka perpindahan bernilai nol.Jadi usaha yang dilakukan si cucu adalah nol.Jawaban yang benar adalah A.

2. Soal Olimpiade Sains Nasional fisika SMP Tingkat Kabupaten/Kota Tahun 2010 No.12Seorang petugas kebersihan mendorong lemari di atas lantai dengan gaya 350 N. Lantai memberikangaya gesek sebesar 70 N. Usaha yang dilakukan untuk memindahkan lemari tersebut sejauh 6 meteradalah : A. 45 J B. 72 JC. 1680 JD. 2580 JPembahasanDiketahui :Gaya dorong (F) = 350 NewtonGaya gesek (Fgesek) = 70 NewtonPerpindahan lemari (s) = 6 meterDitanya : Usaha (W) Jawab :Terdapat dua gaya yang bekerja pada benda yakni gaya dorong (F) dan gaya gesek (Fgesek). Gaya dorong searah dengan perpindahan lemari karenanya gaya dorong melakukan usaha positif.Sebaliknya gaya gesek berlawanan arah dengan perpindahan lemari karenanya gaya gesek melakukanusaha negatif.Usaha yang dilakukan gaya dorong :W = F s = (350 Newton)(6 meter) = 2100 Newton meter = 2100 JouleUsaha yang dilakukan gaya gesek :W = - (Fgesek)(s) = - (70 Newton)(6 meter) = - 420 Newton meter = - 420 JouleUsaha total :W total = 2100 Joule – 420 JouleW total = 1680 JouleJawaban yang benar adalah C.

© 2015 Gurumuda.net | A. San Lohat, S.Pd. 15

Page 18: osn fisika smp

Pembahasan Soal OSN Fisika SMP Tingkat Kabupaten/Kota Gurumuda.Net

Teorema usaha-energi kinetik3. Soal Olimpiade Sains Nasional fisika SMP Tingkat Kabupaten/Kota Tahun 2010 No.10Suatu gaya konstan yang besarnya 8 N bekerja pada benda bermassa 16 kg. Jika benda pada awalnyadiam, maka laju benda setelah gaya tersebut bekerja selama 4 detik adalah....A. 0,5 m/sB. 2 m/sC. 4 m/sD. 8 m/sPembahasanDiketahui :Gaya konstan (F) = 8 NewtonMassa benda (m) = 16 kgKelajuan awal benda (vo) = 0 m/sSelang waktu gaya bekerja pada benda (t) = 4 detikDitanya : Kelajuan akhir benda (vt)Jawab :Usaha = Perubahan energi kinetikW = EK akhir – EK awal W = ½ m vt

2 – ½ m vo2

W = ½ m vt2 – 0

W = ½ m vt2 ------ Persamaan 1

Usaha = Gaya x Perpindahan W = F s W = 8 s Gunakan rumus GLBB untuk menghitung perpindahan (s) s = vo t + ½ a t2 Keterangan : s = perpindahan, vo = kelajuan awal, t = waktu, a = percepatan s = 0 + ½ a t2 = ½ a t2 ----> a = (vt – vo) / t = vt / t s = ½ (vt / t) t2

s = ½ (vt) t Gantikan perpindahan (s) pada persamaan usaha dengan perpindahan (s) pada persamaan ini :W = 8 s W = 8(1/2)(vt)(t) W = (4)(vt)(t) ------ Persamaan 2

Persamaan 1 = Persamaan 2W = W½ m vt

2 = (4)(vt)(t) ½ m vt

= (4)(t) ½ (16)(vt) = 4(4)8 vt = 16vt = 16 / 8vt = 2 meter/sekon Jawaban yang benar adalah B.

4. Soal Olimpiade Sains Nasional fisika SMP Tingkat Kabupaten/Kota Tahun 2011 No.10Untuk meningkatkan laju sebuah benda menjadi 2 kali lajunya semula diperlukan usaha sebesar....

© 2015 Gurumuda.net | A. San Lohat, S.Pd. 16

Page 19: osn fisika smp

Pembahasan Soal OSN Fisika SMP Tingkat Kabupaten/Kota Gurumuda.Net

A. 4 kali energi kinetiknya mula-mulaB. 3 kali energi kinetiknya mula-mulaC. 2 kali energi kinetiknya mula-mulaD. 1 kali energi kinetiknya mula-mulaPembahasanDiketahui :Andaikan massa benda (m) = 1 kgAndaikan kelajuan awal (vo) = 1 m/sKelajuan akhir (vt) = 2 x kelajuan awal = 2 x 1 = 2 m/sDitanya : Usaha = ... ?Jawab :Energi kinetik awal :EK awal = ½ m vo

2 = ½ (1)(1)2 = ½ (1)(1) = ½ (1) = 0,5 Energi kinetik akhir ketika laju benda menjadi 2 kali lajunya semula :EK akhir = ½ m vt

2 = ½ (1)(2)2 = ½ (4) = 2

Teorema usaha-energi kinetik :Usaha = Perubahan energi kinetikUsaha = Energi kinetik akhir – Energi kinetik awalUsaha = 2 – 0,5Usaha = 1,5

Energi kinetik mula-mula = 0,5Usaha = 3 x 0,5 = 1,5Jadi untuk meningkatkan laju sebuah benda menjadi 2 kali lajunya semula diperlukan usaha sebesar 3kali energi kinetiknya mula-mula.Jawaban yang benar adalah B.

Hukum kekekalan energi mekanik5. Soal Olimpiade Sains Nasional SMP Tingkat Kabupaten/Kota Tahun 2011 No.15Sebuah benda memiliki energi mekanik tetap, energi kinetik semakin besar, dan energi potensialgravitasi semakin kecil. Benda tersebut dalam keadaan....A. diamB. bergerak ke atasC. bergerak ke bawahD. bergerak ke atas dipercepatPembahasanEnergi kinetik semakin besar artinya kecepatan benda semakin besar dan energi potensial gravitasisemakin kecil artinya ketinggian benda dari permukaan tanah semakin kecil. Hal ini terjadi ketikabenda bergerak dari ketinggian tertentu ke bawah menuju permukaan tanah.

Hubungan antara kecepatan dan energi kinetik ditunjukkan oleh rumus energi kinetik :EK = ½ m v2

Keterangan rumus : EK = energi kinetik, m = massa, v = kecepatan

Hubungan antara ketinggian dan energi potensial gravitasi ditunjukkan oleh rumus energi potensialgravitasi :EP = m g h

© 2015 Gurumuda.net | A. San Lohat, S.Pd. 17

Page 20: osn fisika smp

Pembahasan Soal OSN Fisika SMP Tingkat Kabupaten/Kota Gurumuda.Net

Keterangan rumus : EP = energi potensial gravitasi, m = massa, g = percepatan gravitasi, h =ketinggianJawaban yang benar adalah C.

Pesawat sederhana 6. Soal Olimpiade Sains Nasional fisika SMP Tingkat Kabupaten/Kota Tahun 2010 No.11Perbandingan panjang a dan b pada gambar skema pesawat sederhana adalah a : b = 1 : 2,2. Nilaiminimum gaya F untuk mengangkat beban B yang beratnya 330 Newton adalah.... NewtonA. 100B. 150C. 2200D. 7260PembahasanDiketahui :Panjang a = lengan beban (lB) = 1Panjang b = lengan gaya (lF) = 2,2 Gaya berat beban (w) = 330 NewtonDitanya : Gaya ungkit (F)Jawab :Rumus keuntungan mekanis pesawat sederhana pengungkit :KM = w / F = lF / lB

Keterangan :KM = keuntungan mekanis, w = gaya berat, F = gaya ungkit, lF = lengan gaya, lB = lengan beban

w / F = lF / lB

330 / F = 2,2 / 1330 / F = 2,2330 / 2,2 = FF = 150 NewtonJawaban yang benar adalah B.

7. Soal Olimpiade Sains Nasional fisika SMP Tingkat Kabupaten/Kota Tahun 2011 No.17Perhatikan gambar di samping ini. Seorang siswa memindahkan benda dari titik A ke titik B dengansebuah gaya F seperti pada gambar. Jika gesekan antara benda dengan papan diabaikan, makakeuntungan mekanik kegiatan itu adalah....A. 4B. 5/3C. 5/4D. 3/4PembahasanDiketahui :Ketinggian bidang miring (y) = 4 meter – 1 meter =3 meterPanjang bidang datar (x) = 4 meter Ditanya : Keuntungan mekanik (KM) bidang miring Jawab :Hitung panjang bidang miring menggunakan rumus Pythagoras :R = √32+42 = √9+16 = √25 = 5 meter

© 2015 Gurumuda.net | A. San Lohat, S.Pd. 18

Page 21: osn fisika smp

Pembahasan Soal OSN Fisika SMP Tingkat Kabupaten/Kota Gurumuda.Net

Keuntungan mekanik bidang miring :KM = R / yKM = 5 meter / 3 meterKM = 5/3Jawaban yang benar adalah B.

Energi Potensial Pegas8. Soal Olimpiade Sains Nasional SMP Tingkat Kabupaten/Kota Tahun 2011 No.18Sebuah arena loncat indah menggunakan alas berpegas sebagai pemberi gaya dorong ke atas terhadapterhadap atlit untuk memulai loncatan. Pada saat seorang atlit melompat ke atasnya dengan gaya 500 N,pegas memendek sepanjang 4 cm. Besar energi potensial yang diberikan pegas untuk mendorong atlititu adalah....A. 20 jouleB. 10 jouleC. 5 jouleD. 2 joulePembahasanDiketahui :Gaya (F) = 500 NPerubahan panjang pegas (Δx) = 4 cm = 0,04 mDitanya : Energi potensial pegasJawab :Terlebih dahulu hitung konstanta elastisitas pegas menggunakan rumus hukum Hooke :k = F / Δx = 500 N / 0,04 m = 12500 N/mEnergi potensial elastis pegas adalah :EP = ½ k Δx2 = ½ (12500)(0,04)2 = (6250)(0,0016) EP = 10 Joule Jawaban yang benar adalah B.

© 2015 Gurumuda.net | A. San Lohat, S.Pd. 19

Page 22: osn fisika smp

Pembahasan Soal OSN Fisika SMP Tingkat Kabupaten/Kota

Fluida Statis

Tekanan fluida1. Soal Olimpiade Sains Nasional SMP Tingkat Kabupaten/Kota Tahun 2010 No.13Perhatikan gambar berikut!

Percepatan gravitasi 10 N/kg, jika luas penampang ikan yang tertekan oleh air di atasnya sebesar 6 cm2,maka gaya yang menekan ikan dari air di atasnya adalah....A. 5,1 NB. 85 NC. 8500 ND. 8600 NPembahasanDiketahui :Percepatan gravitasi (g) = 10 N/kgLuas penampang ikan (A) = 6 cm2 = 6 x 10-4 m2

Massa jenis air = 1 gram/cm3 = 1 (10-3 kg) / 10-6 m3 = 103 kg/m3 = 1000 kg/m3 Ketinggian air = 100 cm – 15 cm = 85 cm = 85 x 10-2 meterDitanya : Gaya yang menekan ikan dari air di atasnyaJawab :Rumus tekanan :P = F / AF = P AF = ρ g h AF = (1000)(10)(85 x 10-2)(6 x 10-4 m2)F = (104)(510 x 10-6)F = 510 x 10-2

F = 5,1 NewtonJawaban yang benar adalah A.

Kapilaritas dan tegangan permukaan zat cair2. Soal Olimpiade Sains Nasional SMP Tingkat Kabupaten/Kota Tahun 2010 No.14Semakin kecil tetesan zat cair, bentuk tetesannya semakin mendekati bentuk bola. Untuk suatu nilaivolume tertentu, dapat dibuktikan bahwa bentuk bola memiliki luas permukaan terkecil dibandingkandengan bentuk-bentuk lainnya. Bentuk tetesan zat cair yang mendekati bentuk bola itu berkaitandengan konsep....A. Massa jenisB. Adhesi dan kohesiC. Tegangan permukaanD. Miniskus dan kapilaritasPembahasan Bentuk tetesan zat cair yang mendekati bentuk bola itu berkaitan dengan konsep tegangan permukaan.Secara matematis, tegangan permukaan adalah gaya per satuan panjang. Gaya yang dimaksud adalah

© 2015 Gurumuda.net | A. San Lohat, S.Pd. 20

Page 23: osn fisika smp

Pembahasan Soal OSN Fisika SMP Tingkat Kabupaten/Kota

gaya kohesi. Kohesi adalah gaya tarik menarik antara molekul sejenis sedangkan gaya adhesi adalahgaya tarik antara molekul tak sejenis. Adanya gaya kohesi yang lebih kuat antara molekul air dengan molekul air dan gaya adhesi yang lemahantara molekul air dengan molekul udara atau molekul benda lainnya menyebabkan semua molekul airsaling tarik menarik untuk berkumpul. Molekul-molekul air di bagian dalam tetesan air, menarikmolekul air di sebelah luar untuk merapat. Molekul air di sebelah luar mempunyai gaya adhesi yanglebih lemah sehingga mudah ditarik untuk merapat. Dan bentuk tetesan air yang terbentuk menyerupaibola. Pada proses ini permukaan tetesan air tampak menegang akibat adanya gaya tarik antara molekulair. Jawaban yang benar adalah C.

3. Soal Olimpiade Sains Nasional SMP Tingkat Kabupaten/Kota Tahun 2010 No.15Peristiwa naiknya air pada pipa kapiler adalah karena....A. gaya adhesi antara air dan kaca mampu menarik air ke atasB. gaya kohesi antara air dan kaca mampu menarik air ke atasC. adanya gaya ArchimedesD. tekanan hidrostatik di permukaan air pada pipa kapiler sangat kecilPembahasanPeristiwa naiknya air pada pipa kapiler terjadi karena gaya adhesi antara molekul air dan molekul pipakapiler lebih besar daripada gaya kohesi antara molekul air dengan molekul air. Gaya adhesi antaramolekul air dan molekul pipa kapiler lebih besar sehingga mampu menarik air ke atas. Pipa kapiler bisa berupa pipa kaca, sumbu kompor, dll. Jawaban yang benar adalah A.

Gaya apung dan hukum Archimedes4. Soal Olimpiade Sains Nasional SMP Tingkat Kabupaten/Kota Tahun 2010 No.16Sebuah kapal yang berlayar di laut memasuki sungai yang cukup lebar dan dalam. Jika massa jenis airlaut 1100 kg/m3, massa jenis air tawar 1000 kg/m3, maka perbandingan gaya Archimedes yang dialamikapal di laut dan di sungai adalah....A. 11 : 10B. 10 : 11C. 121 : 100D. 1 : 1PembahasanDiketahui :Massa jenis air laut (ρ1) = 1100 kg/m3

Massa jenis air tawar (ρ1) = 1000 kg/m3

Ditanya : Perbandingan gaya Achimedes yang dialami kapal di laut dan di sungai Jawab :Jika benda terapung maka gaya apung (FA) = gaya berat (w) :FA = wρair g Vbenda tercelup = ρbenda g Vbenda total

ρair Vbenda tercelup = ρbenda Vbenda total

V bendatercelup

V benda total

=ρbenda

ρair

V benda tercelup=ρbenda

ρair

xV benda total

© 2015 Gurumuda.net | A. San Lohat, S.Pd. 21

Page 24: osn fisika smp

Pembahasan Soal OSN Fisika SMP Tingkat Kabupaten/Kota

Hukum Archimedes menyatakan bahwa gaya apung yang bekerja pada benda yang tercelup dalamfluida (air) sama dengan berat fluida (air) yang dipindahkannya. Volume benda yang tercelup dalamfluida (air) sama dengan volume fluida (air) yang dipindahkan.

Perbandingan volume benda tercelup dalam air laut dan air sungai :V benda tercelupdilaut :V bendatercelup di sungai

ρbenda

ρairlaut

x V bendatotal :ρbenda

ρair sungai

xV benda total

1ρairlaut

:1

ρair sungai

11100

:1

1000111

:1

1010 :11

Jawaban yang benar adalah B.

5. Soal Olimpiade Sains Nasional SMP Tingkat Kabupaten/Kota Tahun 2011 No.16Sebuah balok kayu dicelupkan dalam air. Ternyata 25% bagian berada di atas permukaan air dan 75%berada dalam air. Bila massa jenis air 1 g/cm3 dan percepatan gravitasi bumi 10 m/s2 maka massa jenisbalok kayu adalah.... A. 0,025 kg/m3

B. 0,075 g/cm3

C. 0,250 kg/m3

D. 0,750 g/cm3

PembahasanDiketahui :Bagian balok yang berada di atas permukaan air = 25% = 25/100 = 0,25Bagian balok yang tercelup dalam air = 75% = 75/100 = 0,75Massa jenis air (ρair) = 1 g/cm3 = 1000 kg/m3 Percepatan gravitasi = 10 m/s2

Ditanya : Massa jenis balok kayu (ρbalok) Jawab :Volume balok yang tercelup di dalam air = perbandingan massa jenis balok terhadap massa jenis air.V = ρbalok / ρair

0,75 = ρbalok / 1000ρbalok = (0,75)(1000)ρbalok = 750 kg/m3

ρbalok = (750)(103 g / 106 cm3 = (750)(103)(10-6) g/cm3

ρbalok = 750 x 10-3 g/cm3

ρbalok = 0,750 g/cm3

Jawaban yang benar adalah D.

© 2015 Gurumuda.net | A. San Lohat, S.Pd. 22

Page 25: osn fisika smp

Pembahasan Soal OSN Fisika SMP Tingkat Kabupaten/Kota

Gelombang Bunyi

Intensitas bunyi1. Soal Olimpiade Sains Nasional SMP Tingkat Kabupaten/Kota Tahun 2010 No.20Tingkat intensitas suatu bunyi menyatakan nilai logaritma dari....A. Perbandingan antara daya di suatu tempat dengan daya yang dihasilkan oleh suatu sumber bunyiB. Intensitas suatu bunyi per satuan luas permukaan C. Intensitas suatu bunyi pada jarak tertentu dari sumbernyaD. Perbandingan antara intensitas suatu bunyi dengan intensitas ambang pendengaran manusiaPembahasanRumus tingkat intensitas bunyi :

TI = 10 log II o

Keterangan rumus : TI = tingkat intensitas, I = intensitas suatu bunyi, Io = intensitas ambangpendengaran manusia = 10-12 W/m2

Berdasarkan rumus di atas dapat disimpulkan tingkat intensitas suatu bunyi menyatakan nilai logaritmadari perbandingan antara intensitas suatu bunyi dengan intensitas ambang pendengaran manusia. Jawaban yang benar adalah D.

2. Soal Olimpiade Sains Nasional SMP Tingkat Kabupaten/Kota Tahun 2011 No.22Intensitas suatu bunyi adalah 6 x 10-6 W/cm2. Jika kemudian taraf intensitasnya dinaikkan sebesar 10dB, maka intensitasnya menjadi....A. 6 x 10-5 W/cm2 B. 6 x 10-7 W/cm2

C. 3 x 10-6 W/cm2

D. 12 x 10-6 W/cm2

PembahasanDiketahui :Intensitas (I) = 6 x 10-6 W/cm2

Intensitas ambang (Io) = 10-12 W/m2 = 10-12 W / 104 cm2 = 10-16 W/cm2

Ditanya : Jika TI dinaikkan 10 dB maka I menjadi....Jawab :Penambahan intensitas bunyi sebesar 10 W/m2 = 10-3 W/cm2 setara dengan penambahan taraf intensitasbunyi sebesar 10 dB. Penambahan intensitas bunyi sebesar 102 W/m2 = 10-2 W/cm2 setara denganpenambahan taraf intensitas bunyi sebesar 20 dB. Demikian seterusnya.

Jika kemudian taraf intensitasnya dinaikkan sebesar 10 dB, maka intensitasnya bertambah 10-3 W/cm2.Jadi intensitasnya menjadi (6 x 10-6 W/cm2) + (10-4 W/cm2) = (6 x 10-6 W/cm2) + (100 x 10-6 W/cm2) =106 x 10-6 W/cm2. Tidak ada jawaban yang benar.

Resonansi gelombang bunyi3. Soal Olimpiade Sains Nasional SMP Tingkat Kabupaten/Kota Tahun 2010 No.17Perhatikan gambar berikut!

© 2015 Gurumuda.net | A. San Lohat, S.Pd. 23

Page 26: osn fisika smp

Pembahasan Soal OSN Fisika SMP Tingkat Kabupaten/Kota

Setelah garpu tala A digetarkan oleh pemukul, garpu tala B ikut bergetar. Peristiwa tersebut terjadikarena....A. Garpu tala A memiliki amplitudo sama dengan garpu tala BB. Kedua garpu tala menggunakan kotak resonansi yang samaC. Garpu tala B mempunyai frekuensi sama dengan garpu tala AD. Warna bunyi yang ditimbulkan kedua garpu tala sama.PembahasanSetelah digetarkan oleh pemukul, garpu tala A menggetarkan udara di sekitarnya sehingga udara jugabergetar dengan frekuensi yang sama. Selanjutnya udara menggetarkan garputala B. Garputala B padadasarnya juga sedang bergetar dengan frekuensi alami tetapi amplitudo getarannya sangat kecil dangetarannya tidak dapat dilihat oleh mata. Karena frekuensi alami garputala B sama dengan frekuensigetaran udara dan frekuensi garputala A maka molekul udara memperbesar amplitudo getaran garputalaB. Garputala B tampak bergetar karena amplitudo getarannya dapat dilihat oleh mata. Jawaban yang benar adalah C.

4. Soal Olimpiade Sains Nasional SMP Tingkat Kabupaten/Kota Tahun 2011 No.21Sebuah tabung resonansi menghasilkan suara dengungan keras untuk pertama kalinya pada saatpanjang kolom udara 17 cm dan suara dengungan keras untuk kedua kalinya pada saat panjang kolomudara 51 cm. Bila frekuensi garpu tala yang digunakan adalah 500 Hz. Cepat rambat udara di dalamtabung tersebut adalah....A. 138 m/sB. 230 m/sC. 340 m/sD. 461 m/sPembahasanBentuk tabung resonansi adalah salah satu ujungnya terbuka dan ujung lainnya tertutup sehingga jarakantara titik simpul dan titik perut berurutan = panjang tabung minimum = ¼ λ = λ/4. Rumus panjang tabung :

L=nλ4

→ n=1,3,5,7

Jarak antara titik simpul dan titik perut pada tabung resonansi yang berujung terbuka dan tertutupterpenuhi ketika n berupa bilangan ganjil.

Panjang gelombang untuk n1 = 1 dan L = 0,17 meter :

λ=L 4n

=(0,17 ) 4

1=

0,681

λ=0,68Cepat rambat gelombang :

v=λ f =(0,68 ) (500 )=340 m /sPanjang gelombang untuk n2 = 3 dan L = 0,51 meter :

λ=L 4n

=(0,51 ) 4

3=

2,043

λ=0,68

© 2015 Gurumuda.net | A. San Lohat, S.Pd. 24

Page 27: osn fisika smp

Pembahasan Soal OSN Fisika SMP Tingkat Kabupaten/Kota

Cepat rambat gelombang :v=λ f =(0,68 ) (500 )=340 m /s

Jawaban yang benar adalah C.

Cepat rambat gelombang bunyi5. Soal Olimpiade Sains Nasional SMP Tingkat Kabupaten/Kota Tahun 2010 No.19Untuk mengukur kedalaman laut, sebuah kapal memancarkan gelombang ultrasonik ke dasar laut.Pantulan gelombang tersebut diterima oleh detektor di kapal 12 sekon kemudian. Jika kecepatangelombang ultrasonik di dalam air adalah 1200 m/s, maka dapat disimpulkan kedalaman laut tersebutadalah....A. 7,2 kmB. 14,4 kmC. 21,6 kmD. 28,8 kmPembahasanDiketahui :Pantulan gelombang tersebut diterima oleh detektor di kapal 12 sekon. Jadi 12 sekon adalah waktupergi pulang.Selang waktu (t) = 12 sekon / 2 = 6 sekonKecepatan gelombang ultrasonik di dalam air (v) = 1200 m/sDitanya : Kedalaman laut (s)Jawab :Kedalaman laut (s) :s = v t s = (1200 meter/sekon)(6 sekon) = 7200 meter = 7,2 kilometerJawaban yang benar adalah A.

6. Soal Olimpiade Sains Nasional SMP Tingkat Kabupaten/Kota Tahun 2010 No.22Suara halilintar terdengar 8 detik setelah terlihatnya sebuah kilat. Jika kecepatan cahaya 3 x 108 m/detdan kecepatan bunyi di udara 340 m/det, maka jarak antara sumber halilintar dengan pendengar dapatdiperkirakan sama dengan...A. 1,82 kmB. 2,72 kmC. 4,25 kmD. 7,24 kmPembahasanDiketahui :Selang waktu (t) gerak bunyi = 8 detikKecepatan cahaya (v cahaya) = 3 x 108 meter/detikKecepatan bunyi di udara (v bunyi) = 340 meter/detik Ditanya : Jarak antara sumber halilintar dengan pendengarJawab :Kecepatan cahaya sangat besar sehingga terjadinya kilat dan terlihatnya kilat dapat dianggap terjadipada waktu bersamaan.Jarak sumber halilintar ke pendengar :s = v t = (340 meter/detik)(8 detik) = 2720 meter = 2,72 kmJawaban yang benar adalah B.

© 2015 Gurumuda.net | A. San Lohat, S.Pd. 25

Page 28: osn fisika smp

Pembahasan Soal OSN Fisika SMP Tingkat Kabupaten/Kota

7. Soal Olimpiade Sains Nasional SMP Tingkat Kabupaten/Kota Tahun 2011 No.28Di sebuah tempat seorang pengamat mendengar suara halilintar 16,2 detik setelah penampakan kilat.Jika kecepatan cahaya di udara 3 x 108 m/s dan cepat rambat bunyi di udara saat itu 343 m/s, makadapat ditaksir jarak antara lokasi petir dan pengamat mendekati....A. 5,5 kmB. 12,5 kmC. 25 kmD. 55 kmPembahasanDiketahui :Selang waktu (t) gerak bunyi = 16,2 detikKecepatan cahaya (v cahaya) = 3 x 108 meter/detikKecepatan bunyi di udara (v bunyi) = 343 meter/detik Ditanya : Jarak antara lokasi petir dan pengamatJawab :Kecepatan cahaya sangat besar sehingga terjadinya kilat dan terlihatnya kilat dapat dianggap terjadipada waktu bersamaan.Jarak sumber halilintar ke pendengar :s = v t = (343 meter/detik)(16,2 detik) = 5556,6 meter = 5,5 kmJawaban yang benar adalah A.

Frekuensi gelombang bunyi8. Soal Olimpiade Sains Nasional SMP Tingkat Kabupaten/Kota Tahun 2010 No.24Nada C yang dihasilkan oleh senar gitar dan nada C yang dihasilkan oleh piano memiliki kesamaandalam hal....A. frekuensi bunyiB. warna bunyiC. amplitudo bunyiD. intensitas bunyiPembahasanNada C senar gitar dan nada C piano memiliki kesamaan dalam hal frekuensi bunyi atau nada bunyi.Jawaban yang benar adalah A.

Perambatan gelombang bunyi9. Soal Olimpiade Sains Nasional SMP Tingkat Kabupaten/Kota Tahun 2010 No.25Pada siang hari udara yang berada lebih dekat ke tanah memiliki suhu yang lebih tinggi dari suhu udarayang berada lebih jauh dari tanah. Pada malam hari terjadi sebaliknya, udara yang berada lebih dekat ketanah memiliki suhu yang lebih rendah dari suhu udara yang berada lebih jauh dari tanah. Karena itu....A. Siang hari bunyi lebih banyak merambat ke atas daripada ke arah mendatarB. Malam hari bunyi lebih banyak merambat ke atas daripada ke arah mendatarC. Siang maupun malam hari bunyi merambat ke arah mana sajaD. Siang maupun malam hari bunyi lebih banyak merambat ke arah mendatar daripada ke atasPembahasanPada siang hari, udara yang berada lebih dekat ke tanah memiliki suhu yang lebih tinggi dari suhuudara yang berada lebih jauh dari tanah. Dengan kata lain, pada siang hari udara yang berada lebihdekat ke tanah lebih panas daripada udara yang berada lebih jauh dari tanah. Udara yang lebih panasmempunyai massa jenis lebih kecil sehingga mudah bergerak ke atas. Jadi pada siang hari udara lebih

© 2015 Gurumuda.net | A. San Lohat, S.Pd. 26

Page 29: osn fisika smp

Pembahasan Soal OSN Fisika SMP Tingkat Kabupaten/Kota

banyak bergerak ke atas. Bunyi merambat melalui udara dan bunyi merambat ke arah mana saja.Walaupun demikian, karena pada siang hari udara lebih banyak bergerak ke atas maka bunyi juga lebihbanyak merambat ke atas.

Sebaliknya pada malam hari udara tidak banyak bergerak ke atas sehingga bunyi juga tidak banyakmerambat ke atas. Hal ini yang menjadi alasan mengapa pada malam hari bunyi dari jarak jauh dapatdidengar sedangkan pada siang hari bunyi dari jarak jauh sulit didengar.Jawaban yang benar adalah A.

Efek Doppler10. Soal Olimpiade Sains Nasional fisika SMP Tingkat Kabupaten/Kota Tahun 2011 No.20Sebuah sumber sinyal mengeluarkan sinyal tiap 0,2 detik. Sinyal itu bergerak di udara dengankecepatan 330 m/s. Jika seorang pengamat bergerak menjauhi sumber sinyal dengan kecepatan 30 m/s,maka pengamat menerima sinyal tiap....A. 0,22 detikB. 0,20 detikC. 0,18 detikD. 0,16 detikPembahasanAturan tanda :v selalu positifvp positif jika pendengar mendekati sumber bunyivp negatif jika pendengar menjauhi sumber bunyivs positif jika sumber bunyi menjauhi pendengarvs negatif jika sumber bunyi mendekati pendengarvp = 0 jika pendengar diamvs = 0 jika sumber bunyi diam

Diketahui :Frekuensi sumber sinyal (f) = 1 / 0,2 detik = 5 Hertz Kecepatan pengamat (vp) = -30 m.s-1 (negatif karena pengamat menjauhi sumber bunyi)Kecepatan sumber sinyal (vs) = 0Cepat rambat sinyal (v) = 330 m.s-1

Ditanya : Frekuensi sinyal yang diterima pengamat (f’)Jawab :Rumus efek Doppler :

f '=f ( v± v p

v ± v s)=f ( v −v p

v+vs)=5( 330 −30

330+0 )=5( 300330 )=5( 100

110 )=( 500110 )=( 50

11 )Frekuensi sinyal yang diterima pengamat adalah 50/11 Hertz atau pengamat menerima sinyal tiap 11/50= 0,22 detik.Jawaban yang benar adalah A.

© 2015 Gurumuda.net | A. San Lohat, S.Pd. 27

Page 30: osn fisika smp

Pembahasan Soal OSN Fisika SMP Tingkat Kabupaten/Kota

Osilasi

Getaran harmonis sederhana1. Soal Olimpiade Sains Nasional SMP Tingkat Kabupaten/Kota Tahun 2010 No.18Sebuah benda bergetar dengan frekuensi 5 Hz dalam arah kanan kiri. Benda mulai bergerak dari posisisetimbang ke arah kanan. Waktu yang diperlukan untuk mencapai titik paling kanan sebelas kaliadalah....A. 2,05B. 2,20C. 2,25D. 2,50PembahasanDiketahui :Frekuensi (f) = jumlah getaran selama 1 sekon = 5 HzPeriode (T) = waktu untuk melakukan satu getaran = 1/f = 1/5 = 0,2 sekonDitanya : Waktu yang diperlukan untuk mencapai titik paling kanan sebelas kali adalah....Jawab :

Pola gerakan benda (1 getaran) : B → C → B → A → B Selama 1 getaran, benda melakukan 4 gerakan yakni B ke C, C ke B, B ke A, A ke B.Waktu yang diperlukan untuk melakukan satu gerakan adalah 0,2 sekon / 4 = 0,05sekon.

Jadi waktu yang diperlukan untuk mencapai titik paling kanan (C) sebelas kali = (10x 0,2 sekon) + 0,05 sekon = 2 sekon + 0,05 sekon = 2,05 sekon.Jawaban yang benar adalah A.

2. Soal Olimpiade Sains Nasional SMP Tingkat Kabupaten/Kota Tahun 2011 No.23Sebuah pegas digantungi beban dan digetarkan. Agar frekuensi getarannya menjadi dua kali lipatfrekuensi getar semula, maka massa beban diubah menjadi....A. dua kali massa beban semulaB. empat kali massa beban semulaC. setengah kali massa beban semulaD. seperempat kali massa beban semulaPembahasanRumus frekuensi getaran pegas :

f =1

2π √ km

Keterangan :f = frekuensi, k = konstanta pegas, m = massa beban

Frekuensi (f) getaran pegas jika k = 1 kali, m = 1 kali:

f =1

2 π √ km

=1

2 π √ 11=

12 π

√1=1

2 πAgar frekuensi (f) getaran pegas menjadi 2 kali maka massa (m) diubah menjadi 0,25 kali atau 1/4 kali:

f =1

2 π √ 10,25

=1

2 π√4=

12 π

2=21

2 πJawaban yang benar adalah D.

© 2015 Gurumuda.net | A. San Lohat, S.Pd. 28

Page 31: osn fisika smp

Pembahasan Soal OSN Fisika SMP Tingkat Kabupaten/Kota

Bandul sederhana1. Soal Olimpiade Sains Nasional SMP Tingkat Kabupaten/Kota Tahun 2010 No.23Dua bandul sederhana berada di dua tempat yang berbeda. Panjang tali bandul ke dua adalah 0,4 kalipanjang tali bandul pertama, dan percepatan gravitasi di tempat bandul kedua adalah 0,9 kalipercepatan gravitasi di tempat bandul pertama. Perbandingan frekuensi bandul pertama denganfrekuensi bandul kedua adalah....A. 2/3B. 3/2C. 4/9D. 9/4PembahasanDiketahui :Panjang tali bandul pertama (l1) = 1Panjang tali bandul kedua (l2) = 0,4 (l1) = 0,4 (1) = 0,4Percepatan gravitasi di tempat bandul pertama (g1) = 1Percepatan gravitasi di tempat bandul kedua (g2) = 0,9 (1) = 0,9Ditanya : Perbandingan frekuensi bandul pertama (f1) dengan frekuensi bandul kedua (f2)Jawab :Frekuensi bandul pertama (f1) :

f 1=1

2 π √ gl=

12 π √ 1

1=

12 π

√1=1

2 π(1 )=

12π

Frekuensi bandul kedua (f2) :

f 2=1

2 π √ 0,90,4

=1

2 π√2,25=

12 π

(1,5 )

Perbandingan frekuensi bandul pertama (f1) dengan frekuensi bandul kedua (f2) :f 1 : f 2

12 π

:1

2 π(1,5 )

1:1 (1,5 )

1:1,5 → dikalikan dengan 22 : 3 atau 2/3Jawaban yang benar adalah A.

2. Soal Olimpiade Sains Nasional SMP Tingkat Kabupaten/Kota Tahun 2011 No.25Sebuah beban digantung pada seutas tali dan diberi simpangan sehingga melakukan gerak harmoniksederhana dengan frekuensi 0,5 Hz. Bila tali diperpanjang menjadi 4 kali panjang semula makaperiodenya adalah....A. ¼ detikB. ½ detikC. 2 detikD. 4 detikPembahasanDiketahui :Frekuensi bandul (f) = 0,5 hzDitanya : Berapa periode (T) bandul jika panjang tali (l) menjadi 4 kali panjang semulaJawab :Periode bandul mula-mula :

© 2015 Gurumuda.net | A. San Lohat, S.Pd. 29

Page 32: osn fisika smp

Pembahasan Soal OSN Fisika SMP Tingkat Kabupaten/Kota

T=1f=

10,5 Hertz

=2 sekon

Panjang tali mula-mula :

T=2 π √ lg

T2 π

=√ lg

T 2

22 π2 =lg

22

4 π2 =l

104

4 π2 =l

10

1

π2 =l

10

(1)(10) = l ( π 2

10=l π2

l=10

π2

Jika tali diperpanjang menjadi 4 kali panjang semula :

l=410

π2=

40

π2

Maka periode bandul adalah :

T=2 π √ lg=2 π √ 40

π2 : g=2 π √ 40π2

1g

T=2π √ 40π2

110

=2π √ 4π2 =2π

T=(2 ) (2 )=4Periode bandul adalah 4 detik.Jawaban yang benar adalah D.

© 2015 Gurumuda.net | A. San Lohat, S.Pd. 30

Page 33: osn fisika smp

Pembahasan Soal OSN Fisika SMP Tingkat Kabupaten/Kota

Gelombang Mekanik

Gelombang transversal1. Soal Olimpiade Sains Nasional SMP Tingkat Kabupaten/Kota Tahun 2010 No.21Jarak antara dua lembah gelombang permukaan air yang berdekatan adalah 20 m. Sebuah bendaterapung di permukaan air sehingga mengalami gerak getaran. Jika waktu untuk menempuh satugetaran adalah 4 detik, maka kecepatan gelombang adalah.... m/sA. 20 B. 15C. 10D. 5PembahasanDiketahui :Panjang gelombang (λ) = 20 meterPeriode (T) = 4 detikDitanya : Kecepatan gelombang (v)Jawab :Rumus kecepatan gelombang :v = λ / T = 20 meter / 4 detik = 5 meter / detikJawaban yang benar adalah D.

2. Soal Olimpiade Sains Nasional SMP Tingkat Kabupaten/Kota Tahun 2011 No.26Dua titik A dan B berada pada seutas tali dan terpisah sejauh 90 cm satu terhadap yang lain. Pada taliitu merambat gelombang transversal, sehingga titik A berada di puncak gelombang, titik B di dasargelombang, dan antara keduanya terdapat dua puncak dan dua dasar gelombang. Jika periodegelombang 0,3 detik, maka cepat rambat gelombang itu adalah....A. 10,8 cm/sB. 18,0 cm/sC. 120,0 cm/sD. 200,0 cm/sPembahasanDiketahui :Jarak AB (l) = 90 cm Periode gelombang (T) = 0,3 detikFrekuensi gelombang (f) = 1/0,3 detik Titik A berada di puncak gelombang, titik B di dasar gelombangDi antara titik A dan B terdapat dua puncak dan dua dasar gelombangDitanya : Cepat rambat gelombang (v = f λ)Jawab :

Berdasarkan gambar disimpulkan bahwa terdapat 2,5 panjang gelombang. Jaraksatu panjang gelombang (λ) = 90 cm / 2,5 = 36 cm

Cepat rambat gelombang :v = f λ = (1/0,3)(36) = 36/0,3 = 120 cm/s.Jawaban yang benar adalah C.

3. Soal Olimpiade Sains Nasional SMP Tingkat Kabupaten/Kota Tahun 2010 No.26

© 2015 Gurumuda.net | A. San Lohat, S.Pd. 31

Page 34: osn fisika smp

Pembahasan Soal OSN Fisika SMP Tingkat Kabupaten/Kota

Grafik berikut menampilkan simpangan dari sebuah titik dalam satu medium sebagai fungsi waktuketika sebuah gelombang lewat melalui medium.

Jika panjang gelombang tersebut 6 meter, maka cepat rambat gelombangnya adalah....A. 3 m/sB. 6 m/sC. 8 m/sD. 12 m/sPembahasanDiketahui :Terdapat 2 panjang gelombang berdasarkan grafik di atas. Jarak 1 panjang gelombang (λ) = 6 meter / 2 = 3 meter.Periode (T) = 0,5 sekonDitanya : Cepat rambat gelombang (v)Jawab :v = f λ = λ / T = 3 meter / 0,5 sekon = 6 meter / sekonJawaban yang benar adalah B.

Gelombang longitudinal4. Soal Olimpiade Sains Nasional SMP Tingkat Kabupaten/Kota Tahun 2010 No.27Dua sumber bunyi dibunyikan bersamaan pada medium dan suhu yang sama. Sumber bunyi pertamaberfrekuensi 640 Hz dan sumber bunyi kedua 1280 Hz. Pernyataan yang benar adalah....A. Kecepatan bunyi pertama lebih besar dari kecepatan bunyi keduaB. Kecepatan bunyi pertama lebih kecil dari kecepatan bunyi keduaC. Kecepatan bunyi pertama dan kedua sama besarD. Panjang gelombang bunyi keduanya sama besarPembahasanKecepatan bunyi bergantung pada medium karenanya jika medium sama maka kecepatan kedua bunyisama. Sebaliknya jika frekuensi besar maka panjang gelombang kecil, demikian juga apabila frekuensikecil maka panjang gelombang besar. Frekuensi kedua bunyi berbeda sehingga panjang gelombangkedua bunyi juga berbeda.Jawaban yang benar adalah C.

Gelombang stasioner (gelombang berdiri5. Soal Olimpiade Sains Nasional SMP Tingkat Kabupaten/Kota Tahun 2011 No.24Seutas dawai yang kedua ujungnya dijepit, menghasilkan nada dasar dengan frekuensi 420 Hz.Frekuensi nada atas ketiganya adalah....A. 840 HzB. 1260 HzC. 1680 HzD. 2940 HzPembahasanDiketahui :Frekuensi nada dasar (f1) = 420 Hz

© 2015 Gurumuda.net | A. San Lohat, S.Pd. 32

Page 35: osn fisika smp

Pembahasan Soal OSN Fisika SMP Tingkat Kabupaten/Kota

Kedua ujung dawai terikatDitanya : Frekuensi nada atas ketiganyaJawab :Frekuensi nada atas kedua (f2) = 2 f1 = 2 (420 Hz) = 840 HzFrekuensi nada atas ketiga (f3) = 3 f1 = 3 (420 Hz) = 1260 HzJawaban yang benar adalah B.

6. Soal Olimpiade Sains Nasional SMP Tingkat Kabupaten/Kota Tahun 2013 No.30Panjang gelombang nada atas pertama pada seutas dawai adalah 40 cm. Bila cepat rambat bunyi diudara 340 m/s, frekuensi nada atas ke tiga dari getaran dawai tersebut adalah....A. 850 HzB. 1600 HzC. 1700 HzD. 3200 HzPembahasanDiketahui :Panjang gelombang nada atas pertama (λ) = 40 cm = 0,4 meterCepat rambat bunyi di udara (v) = 340 meter/sekonDitanya : Frekuensi nada atas ke tiga dari getaran dawaiJawab :Di bawah adalah gambar gelombang berdiri pada dawai yang kedua ujungnya terikat. Terlebih dahuluhitung panjang dawai menggunakan data panjang gelombang nada atas pertama. Setelah itu, sebelummenghitung frekuensi nada atas ketiga, terlebih dahulu hitung panjang gelombang nada atas ketiga.

Panjang gelombang nada ataspertama :Panjang dawai (L) = 2. ½ λ Panjang dawai (L) = λ Panjang dawai (L) = 0,4 meterPanjang gelombang nada atas ketiga :L = 2 λ 0,4 = 2 λλ = 0,4 / 2λ = 0,2 meterFrekuensi nada atas ketiga :f = v / λf = 340 : 0,2 f = 1700 HertzJawaban yang benar adalah C.

© 2015 Gurumuda.net | A. San Lohat, S.Pd. 33

Page 36: osn fisika smp

Pembahasan Soal OSN Fisika SMP Tingkat Kabupaten/Kota

Gelombang bunyi7. Soal Olimpiade Sains Nasional SMP Tingkat Kabupaten/Kota Tahun2011 No.27Pada gambar di bawah ini dilukiskan sebuah bel listrik yang beradadalam sebuah sungkup yang tertutup. Kemudian bel dibunyikan danudara dalam sungkup dikeluarkan. Setelah udara dari sungkup habisdikeluarkan, bunyi bel listrik tidak terdengar lagi. Berdasarkan peristiwaitu dapat disimpulkan bahwa bunyi....A. tidak terjadi dalam ruang tertutupB. terbawa oleh udara keluar sungkupC. tidak dapat merambat dalam ruang hampa udaraD. tidak dapat merambat melalui zat padatPembahasanBunyi merupakan gelombang yang merambat melalui zat padat, cair dan gas. Jika bel berada di dalamruang hampa udara maka gelombang bunyi tidak bisa merambat dari bell ke telinga pendengar.Jawaban yang benar adalah C.

8. Soal Olimpiade Sains Nasional SMP Tingkat Kabupaten/Kota Tahun 2014 No.27Bagian alat musik gitar yang merupakan sumber bunyiadalah....A. kotak udaraB. lubang resonansiC. pasak pengaturD. dawaiPembahasanSemua sumber bunyi adalah benda yang bergetar. Benda yangbergetar pada gitar adalah dawai (senar) karenanya bagian alatmusik gitar yang merupakan sumber bunyi adalah dawai. Jawaban yang benar adalah D.

© 2015 Gurumuda.net | A. San Lohat, S.Pd. 34

Page 37: osn fisika smp

Pembahasan Soal OSN Fisika SMP Tingkat Kabupaten/Kota

Alat Optik Cermin dan Lensa

Cermin datar1. Soal Olimpiade Sains Nasional SMP Tingkat Kabupaten/Kota Tahun 2010 No.34Sebuah lilin berada di depan dua buah cermin datar yang saling membentuk sudut masing-masingsebesar 60o, maka jumlah bayangan lilin tersebut adalah sebanyak....A. 3 buahB. 4 buahC. 5 buahD. 6 buahPembahasan

Amati kedua gambar di samping. Benda (bulatan hijau)berada di depan permukaan cermin yang memantulkancahaya. Sinar yang datang dari benda menuju cermindatar, tegak lurus terhadap permukaan cermin datar aliasmembentuk sudut 90o. Hukum pemantulan cahayamenyatakan bahwa sudut datang sama dengan sudutpantul. Jika sudut datang bernilai 90o maka sudut pantuljuga bernilai 90o.

Pada gambar kiri, benda berada di tengah-tengah keduacermin datar dan bayangan yang terbentuk adalah dua buah. Pada gambar kanan, benda berjarak lebihdekat ke salah satu cermin datar dan bayangan yang terbentuk adalah tiga buah.Jawaban yang benar adalah A.

Cermin cekung2. Soal Olimpiade Sains Nasional SMP Tingkat Kabupaten/Kota Tahun 2010 No.31Seorang dokter gigi mengamati dan memeriksa lubang gigi geraham belakang seorang pasien denganmenggunakan sebuah cermin yang berjari-jari 8 cm, agar lubang gigi pasien tersebut dapat terlihatdengan jelas dan cukup besar oleh dokter, maka gigi pasien harus berada di depan cermin pada jarak....A. kurang dari 4 cm di depan cermin cekungB. kurang dari 4 cm di depan cermin cembungC. lebih dari 4 cm di depan cermin cekungD. lebih dari 4 cm di depan cermin cembungPembahasanDiketahui :Jari-jari cermin (r) = 8 cmPanjang fokus cermin (f) = r / 2 = 8 / 2 = 4 cm Ditanya : Berapa jarak gigi pasien di cermin ?Jawab :Cermin yang digunakan adalah cermin cekung atau cermin cembung ? Pada soal tertulis agar lubanggigi pasien tersebut dapat terlihat dengan jelas dan cukup besar oleh dokter karenanya cermin yangdigunakan harus bisa memperbesar bayangan gigi pasien dan bayangan itu harus tegak. Cermincembung selalu menghasilkan bayangan terbalik dan ukuran bayangan lebih kecil daripada ukuranbenda. Sebaliknya cermin cekung dapat menghasilkan bayangan tegak jika jarak benda (s) lebih kecildaripada panjang fokus (f). Apabila jarak benda lebih besar daripada panjang fokus (f) maka cermincekung menghasilkan bayangan terbalik.

© 2015 Gurumuda.net | A. San Lohat, S.Pd. 35

Page 38: osn fisika smp

Pembahasan Soal OSN Fisika SMP Tingkat Kabupaten/Kota

Panjang fokus (f) cermin cekung adalah 4 cm karenanya gigi pasien harus berjarak kurang dari 4 cm didepan cermin cekung. Jawaban yang benar adalah A.

3. Soal Olimpiade Sains Nasional SMP Tingkat Kabupaten/Kota Tahun 2011 No.33Suatu cermin cekung mempunyai jari-jari kelengkungan 24 cm. Jika benda diletakkan 20 cm di depancermin tersebut maka bayangan yang dihasilkan bersifat....A. nyata, tegak dan diperbesarB. nyata, terbalik dan diperbesarC. maya, tegak dan diperbesarD. maya, terbalik dan diperkecilPembahasanDiketahui :Jari-jari kelengkungan (r) = 24 cmPanjang fokus (f) = R/2 = 24/2 = +12 cmPanjang fokus cermin cekung bertanda positif atau bersifat nyata karena berkas cahaya melewati titikfokus cermin cekung.Jarak benda (s) = 20 cm Ditanya : Sifat bayanganJawab :Bayangan bersifat maya atau nyata ? Untuk mengetahuinya terlebih dahulu hitung jarak bayangan (s’) :1/s + 1/s’ = 1/f1/s’ = 1/f – 1/s1/s’ = 1/12 – 1/201/s’ = 5/60 – 3/601/s’ = 2/60s’ = 60/2 s’ = 30 cmJarak bayangan bertanda positif artinya bayangan bersifat nyata karena dilewati cahaya (bandingkandengan gambar di samping).

Bayangan diperbesar atau diperkecil ? Bayangan tegak atau terbalik ? Untuk mengetahuinya terlebihdahulu hitung perbesaran bayangan (M) :M = -s’ / s = -30/20 = -1,5M > 1 artinya bayangan diperbesar, M bertanda negatif artinya bayangan terbalik (bandingkan dengangambar di samping). Jadi sifat-sifat bayangan adalah nyata, terbalik, diperbesarJawaban yang benar adalah B.

Cermin cembung4. Soal Olimpiade Sains Nasional SMP Tingkat Kabupaten/Kota Tahun 2010 No.35Cahaya yang jatuh ke cermin cembung dengan arah sejajar sumbu utama cermin akan dipantulkan....A. menuju titik fokus cerminB. seolah-olah dari titik fokus cerminC. melalui titik pusat kelengkungan cerminD. tegak lurus terhadap bidang cerminPembahasanAmati gambar di bawah.

© 2015 Gurumuda.net | A. San Lohat, S.Pd. 36

Page 39: osn fisika smp

Pembahasan Soal OSN Fisika SMP Tingkat Kabupaten/Kota

Jawaban yang benar adalah B.

5. Soal Olimpiade Sains Nasional SMP Tingkat Kabupaten/Kota Tahun 2011 No.32Seorang pengendara motor melihat bayangan kendaraan di belakangnya 1/6 kali ukuran sebenarnya.Bila jarak pengemudi tersebut ke kendaraan yang di belakangnya itu 30 meter, berarti spion motor itumenggunakan cermin dengan jari-jari kelengkungan....A. 7,14 mB. 8,57 mC. 12,00 mD. 24,00 mPembahasanDiketahui :Perbesaran bayangan (M) = 1/6 kaliJarak benda (s) = 30 meterDitanya : Jari-jari kelengkungan cermin cembung spion (R)Jawab :Hitung jarak bayangan (s’)Karena telah diketahui perbesaran bayangan (M) dan jarak benda (s) maka jarak bayangan bisadiketahui menggunakan rumus perbesaran bayangan :M = -s / s1/6 = -s / 30s = -30 / 6s = -5 meterTanda negatif artinya bayangan bersifat maya. Bayangan berjarak 5 meter di belakang cermincembung.

Hitung panjang fokus (f)Karena telah diketahui jarak benda (s) dan jarak bayangan (s’) maka panjang fokus bisa dihitungmenggunakan rumus cermin lengkung :1/s + 1/s' = 1/f1/30 + 1/-5 = 1/f1/30 – 1/5 = 1/f1/30 – 6/30 = 1/f-5/30 = 1/ff = -30/5f = -6 meter

Hitung jari-jari kelengkungan (R)Jari-jari kelengkungan cermin cembung adalah dua kali panjang fokus cermin cembung. R = 2 f = 2 (6 meter) = 12 meterJari-jari kelengkungan cermin cembung spion adalah 12 meter.Jawaban yang benar adalah C.

© 2015 Gurumuda.net | A. San Lohat, S.Pd. 37

Page 40: osn fisika smp

Pembahasan Soal OSN Fisika SMP Tingkat Kabupaten/Kota

Lensa cembung6. Soal Olimpiade Sains Nasional SMP Tingkat Kabupaten/Kota Tahun 2010 No.36Sebuah benda nyata dan tegak diletakkan pada jarak 50 cm dari sebuah lensa positif berjarak fokus 25cm. Sifat bayangan yang dihasilkan adalah.....A. nyata, terbalik dan diperbesarB. maya, tegak dan sama besar dengan bendanyaC. maya, tegak dan diperbesarD. nyata, terbalik dan sama besar dengan bendanyaPembahasanDiketahui :Jarak benda (s) = 50 cmPanjang fokus lensa (f) = +25 cmPanjang fokus bertanda positif karena lensa yang digunakan adalah lensa positif atau lensa cembung.Ditanya : Sifat bayangan adalah.....Jawab :Bayangan bersifat maya atau nyata ? Untuk mengetahuinya, terlebih dahulu hitung jarak bayangan :1/s’ = 1/f – 1/s1/s’ = 1/25 – 1/501/s’ = 2/50 – 1/501/s’ = 1/50s’ = 50/1s’ = 50 cmJarak bayangan bertanda positif artinya bayangan bersifat nyata karena dilewati cahaya.

Bayangan diperbesar atau diperkecil ? Bayangan bersifat tegak atau terbalik ? Untuk mengetahuinyaterlebih dahulu hitung perbesaran bayangan :M = -s’ / s = -50/50 = -1M = 1 artinya bayangan sama besar dengan bendaM bertanda negatif artinya bayangan bersifat terbalik

Jadi sifat bayangan adalah nyata, terbalik dan sama besar dengan bendanya.Jawaban yang benar adalah D.

© 2015 Gurumuda.net | A. San Lohat, S.Pd. 38

Page 41: osn fisika smp

Pembahasan Soal OSN Fisika SMP Tingkat Kabupaten/Kota

Alat Optik Mata, Kacamata, Teropong

Mata1. Soal Olimpiade Sains Nasional SMP Tingkat Kabupaten/Kota Tahun 2010 No.28Salah satu alat optik alami adalah mata. Pada proses melihat, terbentuk bayangan .....A. Sejati, tegak, tepat di retinaB. Sejati, terbalik, tepat di retinaC. Maya, tegak, tepat di retinaD. Maya, terbalik, tepat di retinaPembahasanTitik dekat mata normal rata-rata adalah 25 cm. Titik dekat adalah jarak terdekat yang masih dapatdifokuskan oleh mata. Diameter mata manusia tentu saja lebih kecil dari 25 cm sehingga panjang fokuslensa mata juga lebih kecil dari 25 cm. Dengan demikian, jarak benda (s) pasti lebih besar daripadapanjang fokus sistem lensa mata (f).Sistem lensa mata merupakan lensa cembung. Karenanya bayangan yang terbentuk oleh sistem lensamata pada proses melihat mempunyai sifat yang sama dengan bayangan yang terbentuk oleh lensacembung. Sifat-sifat bayangan yang terbentuk oleh sistem lensa mata ketika jarak benda (s) lebih besardaripada panjang fokus (f), antara lain :- Nyata atau sejati- Terbalik- Diperkecil- Tepat di retinaBayangan harus tepat berada di retina karena retina berfungsi mengubah gelombang cahaya menjadisinyal listrik yang diteruskan ke otak. Walaupun bayangan terbalik tetapi otak manusia mengubahnyamenjadi tegak sehingga benda-benda yang dilihat oleh mata bersifat tegak.Ketika anda melihat pohon dari jarak jauh maka pohon terlihat lebih kecil dari ukurannya yangsebenarnya. Hal ini sesuai dengan sifat bayangan yakni diperkecil.Jawaban yang benar adalah A.

Kacamata2. Soal Olimpiade Sains Nasional SMP Tingkat Kabupaten/Kota Tahun 2011 No.37Seorang mempunyai titik dekat 16 cm dan titik jauh 80 cm. Jika ia menggunakan kacamata, ia dapatmelihat benda-benda yang sangat jauh dengan jelas. Dengan menggunakan kacamata tersebut, jarakbenda terdekat yang dapat dia lihat dengan jelas adalah....A. 13 1/3 cmB. 20 cmC. 36 cmD. 48 1/3 cmPembahasanDiketahui :Titik jauh orang itu hanya 80 cm karenanya dapat disimpulkan orang itu menderita rabun jauh. Rabun jauh terjadi karena sistem lensa mata lebih melengkung atau kurang pipih dari yang seharusnyaseperti pada mata normal sehingga panjang fokus sistem lensa mata berkurang. Hal ini menyebabkanberkas cahaya dari benda berjarak tak berhingga (titik jauh) tidak difokuskan pada retina tetapidifokuskan di depan retina.Ditanya : Jarak benda terdekat yang dapat dilihat dengan jelas menggunakan kacamataJawab :

© 2015 Gurumuda.net | A. San Lohat, S.Pd. 39

Page 42: osn fisika smp

Pembahasan Soal OSN Fisika SMP Tingkat Kabupaten/Kota

Titik jauh orang itu adalah 80 cm. Lensa kacamata harus membentuk bayangan pada jarak 80 cm didepannya. Bayangan berada di depan mata dan lensa kacamata sehingga bayangan bersifat maya dantegak. Jadi jarak bayangan (s’) = -80 cm. Jika orang itu menggunakan kacamata, ia dapat melihat benda-benda yang sangat jauh dengan jelas.Jadi jarak benda (s) = titik jauh mata normal = tak berhingga = ~.

Panjang fokus lensa mata :1/f = 1/s + 1/s’1/f = 1/~ + (- 1/80)1/f = 0 - 1/80 1/f = - 1/80f = - 80 / 1 f = - 80 cmPanjang fokus negatif artinya lensa kacamata yang digunakan adalah lensa cekung atau lensa divergen.

Jika orang itu menggunakan lensa kacamata yang sama maka berapa jarak benda terdekat yang dapatdilihat dengan jelas ? Panjang fokus lensa (f) = -80 cm. Lensa harus membentuk bayangan pada jarak16 cm di depan mata dan lensa sehingga bayangan bersifat maya dan bertanda negatif. Jadi jarakbayangan (s’) = -16 cm. 1/s = 1/f - 1/s’ = -1/80 – (-1/16) = -1/80 + 1/16 = -1/80 + 5/80 = 4/80 s = 80/4 = 20 cm.Jarak benda terdekat yang dapat dia lihat dengan jelas adalah 20 cm.Jawaban yang benar adalah B.

3. Soal Olimpiade Sains Nasional SMP Tingkat Kabupaten/Kota Tahun 2010 No.37Kacamata merupakan alat bantu agar penglihatan menjadi jelas. Seringkali kamu melihat temanmuyang menggunakan kacamata. Bagaimanakah perjalanan berkas sinar yang benar pada lensa yangdigunakan temanmu ?Pembahasan

Lensa kacamata yang digunakanberdasarkan gambar adalah lensacekung. Perjalanan berkas cahaya yangdatang dan dibiaskan lensa cekungharus seperti pada jawaban B.

4. Soal Olimpiade Sains Nasional SMP Tingkat Kabupaten/Kota Tahun 2011 No.36Perhatikan gambar jalannya sinar pada mata sebelum dan sesudah pakai kacamata berikut ini.Berdasarkan gambar itu dapat dipastikan bahwa....

© 2015 Gurumuda.net | A. San Lohat, S.Pd. 40

Page 43: osn fisika smp

Pembahasan Soal OSN Fisika SMP Tingkat Kabupaten/Kota

Cacat mata Lensa kacamataA Hipermetropi NegatifB Hipermetropi Positif C Miopi Negatif D Miopi Positif

Pembahasan

Rabun dekat = hipermetropiLensa konvergen = lensa cembung = lensa positifJawaban yang benar adalah B.

Teropong5. Soal Olimpiade Sains Nasional SMP Tingkat Kabupaten/Kota Tahun 2011 No.38Perbesaran sebuah teropong yang panjangnya 10 cm adalah 3. Panjang fokus obyektif dan panjangfokus okuler berturut-turut adalah....A. 5 cm dan 5 cmB. 7,5 cm dan 2,5 cmC. 2,5 cm dan 7,5 cmD. 4 cm dan 6 cmPembahasanDiketahui :Perbesaran teropong (M) = 3Panjang teropong (l) = 10 cmDitanya : Panjang fokus lensa obyektif dan panjang fokus lensa okulerJawab :Rumus panjang teropong (l) ketika mata berakomodasi minimum :l = fob + fok

10 cm = fob + fok ----- Persamaan 1Rumus perbesaran teropong (M) ketika mata berakomodasi minimum :M = fob / fok 3 = fob / fok fob = 3 fok ----- Persamaan 2Gantikan fob pada persamaan 1 dengan fok pada persamaan 2 :10 cm = fob + fok

10 cm = 3 fok + fok

10 cm = 4 fok fok = 10 cm / 4 fok = 2,5 cm Panjang fokus lensa okuler = 2,5 cmPanjang fokus lensa obyektif = 3 fok = 3 (2,5 cm) = 7,5 cmPanjang fokus lensa obyektif dan panjang fokus lensa okuler berturut-turut adalah 7,5 cm dan 2,5 cm.Jawaban yang benar adalah B.

© 2015 Gurumuda.net | A. San Lohat, S.Pd. 41

Page 44: osn fisika smp

Pembahasan Soal OSN Fisika SMP Tingkat Kabupaten/Kota

Gelombang Cahaya

Karakteristik cahaya1. Soal Olimpiade Sains Nasional SMP Tingkat Kabupaten/Kota Tahun 2010 No.29Cahaya merupakan gelombang elektromagnetik, oleh karena itu cahaya....A. Tidak mengalami pemantulanB. Tidak mengalami pembiasanC. Tidak memerlukan medium perambatanD. Tidak membawa energi dalam penjalarannyaPembahasanCahaya merupakan gelombang karenanya mengalami pemantulan, pembiasan dan membawa energidalam penjalarannya sebagaimana gelombang-gelombang yang lain. Cahaya termasuk gelombangelektromagnetik sehingga tidak memerlukan medium perambatan.Jawaban yang benar adalah C.

2. Soal Olimpiade Sains Nasional SMP Tingkat Kabupaten/Kota Tahun 2010 No.33Perhatikan pernyataan berikut ini :1. Cahaya bergerak menurut garis lurus2. Cahaya merupakan gelombang elektromagnetik3. Cahaya dapat diserap4. Cahaya membawa muatan listrikPernyataan yang benar mengenai sifat-sifat cahaya adalah....A. pernyataan 1 sajaB. pernyataan 1 dan 2C. pernyataan 1, 2 dan 3D. semua pernyataan benarPembahasan1. Cahaya bergerak menurut garis lurus.Pernyataan ini benar karena walaupun cahaya bergerak seperti gelombang transversal tetapi lintasangerak cahaya berupa garis lurus. Pada pokok bahasan optika geometri (pemantulan dan pembiasancahaya), cahaya digambarkan berupa sinar yang lintasan geraknya berupa garis lurus).2. Cahaya merupakan gelombang elektromagnetik :Pernyataan ini benar karena cahaya merupakan gelombang elektromagnetik, di mana perambatancahaya tidak memerlukan medium. 3. Cahaya dapat diserap :Pernyataan ini benar. Cahaya merupakan salah satu jenis gelombang elektromagnetik. Semuagelombang, baik gelombang mekanik maupun gelombang elektromagnetik, selalu membawa energiketika bergerak dari satu tempat ke tempat yang lain. Karenanya sebagian energi cahaya dapat diserapoleh medium yang dilewatinya. Sebagai contoh, ketika cahaya bergerak menuju cermin datar, sebagianbesar cahaya dipantulkan tetapi sebagian kecil cahaya diserap oleh cermin datar.4. Cahaya membawa muatan listrik :Pernyataan ini benar. Cahaya merupakan gelombang elektromagnetik di mana gelombangelektromagnetik terjadi karena adanya osiliasi muatan listrik. Karena gelombang elektromagnetik tidakmembutuhkan medium perambatan maka muatan listrik bukan medium yang dilalui gelombangelektromagnetik, tetapi muatan listrik selalu bergerak bersamaan dengan merambatnya gelombangelektromagnetik. Jawaban yang benar adalah D.

© 2015 Gurumuda.net | A. San Lohat, S.Pd. 42

Page 45: osn fisika smp

Pembahasan Soal OSN Fisika SMP Tingkat Kabupaten/Kota

Pemantulan sempurna3. Soal Olimpiade Sains Nasional SMP Tingkat Kabupaten/Kota Tahun 2010 No.38Gambar di samping adalah batu perhiasan berlian yang sangat mahal harganya karena cahaya yangdipantulkan berlian begitu berkilauan. Peristiwa tersebut disebabkan karena....A. pembiasan sempurna di mana sinar masuk dari medium kurang rapat ke mediumlebih rapatB. pembiasan sempurna di mana sudut datang lebih kecil dari sudut batasC. pemantulan sempurna di mana sinar datang dari medium kurang rapat ke mediumlebih rapat D. pemantulan sempurna di mana sudut datang lebih besar daripada sudut batasPembahasanCahaya berlian tampak berkilauan karena adanya dispersi cahaya dan pemantulan sempurna.Pemantulan sempurna terjadi ketika cahaya bergerak dari medium yang memiliki indeks bias lebihbesar ke medium yang memiliki indeks bias lebih kecil. Misalnya cahaya bergerak dari berlian (indeksbias lebih besar) ke udara (indeks bias lebih kecil). Untuk memperjelas pemahamanmu, amati gambardi bawah.

Ketika cahaya bergerak darimedium berindeks bias besarke medium berindeks biaskecil, cahaya dibelokkanmenjauhi garis normal. Jikasudut datang (i) semakin besar

maka sudut bias (r) semakin besar. Pada sudut datang tertentu, sudut bias bernilai 90o. Sudut datangketika sudut bias bernilai 90o disebut sudut kritis (θk) atau sudut batas. Apabila sudut datang lebih besardari sudut kritis atau sudut batas maka tidak ada cahaya yang dibiaskan; semua cahaya dipantulkan. Jawaban yang benar adalah D.

4. Soal Olimpiade Sains Nasional SMP Tingkat Kabupaten/Kota Tahun 2011 No.39Fenomena berikut ini, yang merupakan gejala pemantulan sempurna adalah....A. pelangiB. fatamorganaC. telaga warnaD. astigmatismaPembahasanFatamorgana disebut gejala pemantulan sempurna karena pada proses tersebut cahaya yang datang dariudara tidak ada yang dibiaskan oleh permukaan jalan tetapi seolah-olah semua cahaya dipantulkan olehpermukaan jalan. Fatamorgana terjadi karena cahaya bergerak dalam lapisan-lapisan udara yangmempunyai indeks bias berbeda karena kerapatan lapisan-lapisan udara tersebut berbeda. Dalamkenyataannya, cahaya tidak dipantulkan oleh permukaan jalan tetapi dibiaskan (dibelokkan) olehlapisan udara yang mempunyai indeks bias yang berbeda-beda.Jawaban yang benar adalah B.

5. Soal Olimpiade Sains Nasional SMP Tingkat Kabupaten/Kota Tahun 2010 No.30Pemandangan seperti lapisan minyak di permukaan jalan aspal ketika hari panas pada dasarnya adalahperistiwa pemantulan total karena....A. indeks bias lapisan-lapisan udara bagian atas lebih besar daripada bagian bawahB. indeks bias lapisan-lapisan udara bagian atas lebih kecil daripada bagian bawahC. titik-titik air memantulkan berbagai warna pada sudut pantul yang berbeda

© 2015 Gurumuda.net | A. San Lohat, S.Pd. 43

Page 46: osn fisika smp

Pembahasan Soal OSN Fisika SMP Tingkat Kabupaten/Kota

D. titik-titik air membiaskan berbagai warna pada sudut bias yang berbedaPembahasanPemandangan seperti lapisan minyak di permukaan jalan aspal pada siang hari yang panas merupakancontoh fatamorgana. Fatamorgana terjadi akibat adanya perbedaan indeks bias lapisan udara yang dekatdengan jalan dan lapisan udara di atasnya. Pada siang hari yang panas, permukaan jalan memanaskan udara yang berada tepat di atasnya sehinggalapisan udara yang berada tepat di atas permukaan jalan lebih panas dibandingkan lapisan udara disebelah atas yang lebih jauh dari permukaan jalan. Lapisan udara yang lebih panas mengalamipemuaian volume lebih besar sehingga udara tersebut kurang rapat (massa jenisnya lebih kecil).Sebaliknya lapisan udara kurang panas di atasnya mengalami pemuaian volume lebih kecil sehinggaudara tersebut lebih rapat (massa jenisnya lebih besar). Jadi lapisan udara bagian bawah kurang rapatsedangkan lapisan udara di bagian atas lebih rapat. Karena lapisan udara di bagian bawah (lebih dekat dengan permukaan jalan) kurang rapat maka cahayabergerak lebih cepat ketika melewatinya, sedangkan lapisan udara di bagian atas lebih rapat sehinggacahaya bergerak lebih lambat ketika melewatinya.Rumus indeks bias adalah : n = c / v , di mana n adalah indeks bias, c = kecepatan cahaya dalam ruanghampa = konstan = 3 x 108 m/s, v = kecepatan cahaya dalam medium. Berdasarkan rumus ini, c tetapsehingga apabila v semakin besar maka n semakin kecil dan sebaliknya jika v berkurang maka nsemakin besar. Cahaya bergerak lebih cepat pada lapisan udara bagian bawah (v besar) sehingga lapisan udara bagianbawah mempunyai indeks bias lebih kecil (n kecil) sebaliknya cahaya bergerak lebih lambat padalapisan udara bagian atas (v kecil) sehingga lapisan udara bagian atas mempunyai indeks bias lebihbesar (n besar). Jawaban yang benar adalah A.

Indeks bias6. Soal Olimpiade Sains Nasional SMP Tingkat Kabupaten/Kota Tahun 2011 No.29Berkas sinar tampak mempunyai panjang gelombang 510 nm di udara. Jika berkas sinar tersebut masukke dalam medium yang indeks biasnya 1,5 maka panjang gelombangnya berubah menjadi....A. 680 nmB. 510 nmC. 340 nmD. 170 nmPembahasanDiketahui :Panjang gelombang (λ) = 510 nm = 510 x 10-9 meterIndeks bias (n) = 1,5Ditanya : Panjang gelombang (λ) berubah menjadi....Jawab :

λn=λn=

510nm1,5

=340nm

Panjang gelombang (λ) berubah menjadi 340 nanometer.Jawaban yang benar adalah C.

7. Soal Olimpiade Sains Nasional SMP Tingkat Kabupaten/Kota Tahun 2010 No.32Terlihat seekor ikan di dasar laut sebuah kolam yang jernih, kedalaman kolam 2 meter, indeks biaskolam 4/3. Jika ikan dilihat secara hampir tegak lurus dari permukaan kolam, berapa jarak ikan yangtampak dari permukaan kolam ?

© 2015 Gurumuda.net | A. San Lohat, S.Pd. 44

Page 47: osn fisika smp

Pembahasan Soal OSN Fisika SMP Tingkat Kabupaten/Kota

A. 1 meter dari permukaan kolamB. 1,5 meter dari permukaan kolamC. 2 meter dari permukaan kolamD. 2,67 meter dari permukaan kolamPembahasanDiketahui :Indeks bias kolam = 4/3 Kedalaman kolam = 2 meterDitanya : Jarak ikan yang tampak dari permukaan kolam ?Jawab :Jarak semu = 1 : n (jarak sesungguhnya)Jarak semu = 1 : 4/3 (2 meter)Jarak semu = 1 x 3/4 (2 meter)Jarak semu = 3/4 (2 meter)Jarak semu = 6/4Jarak semu = 1,5 meterJarak ikan yang tampak dari permukaan kolam = 1,5 meter Jawaban yang benar adalah B.

8. Soal Olimpiade Sains Nasional SMP Tingkat Kabupaten/Kota Tahun 2011 No.30Cahaya dapat ditransmisikan dalam suatu saluran yang indeks bias relatifnya terhadap lingkungan....A. > 1B. = 1C. < 1D. = 0PembahasanIndeks bias hampa udara = 1,0000Indeks bias medium lebih besar dari indeks bias hampa udara.Jawaban yang benar adalah A.

Dispersi cahaya9. Soal Olimpiade Sains Nasional SMP Tingkat Kabupaten/Kota Tahun 2011 No.34Sudut dispersi adalah sudut antara dua warna cahaya monokromatik yang terjadi pada peristiwa....A. polarisasi cahayaB. absorbsi cahayaC. dispersi cahayaD. difraksi cahayaPembahasanCahaya monokromatik adalah cahaya yang mempunyai satu warna. Cahaya putih seperti cahayamatahari merupakan gabungan dari semua warna cahaya di mana masing-masing warna mempunyaipanjang gelombang atau frekuensi yang berbeda-beda. Penguraian cahaya putih menjadi spektrumcahaya tampak (merah, jingga, kuning, hijau, biru, nila, ungu) disebut dispersi cahaya. Dispersi cahaya bisa dilakukan oleh prisma, tetesan air atau benda lain. Pelangi merupakan contohdispersi cahaya dalam kehidupan sehari-hari, di mana cahaya matahari didispersikan oleh tetesan airhujan. Jawaban yang benar adalah C.

© 2015 Gurumuda.net | A. San Lohat, S.Pd. 45

Page 48: osn fisika smp

Pembahasan Soal OSN Fisika SMP Tingkat Kabupaten/Kota

10. Soal Olimpiade Sains Nasional SMP Tingkat Kabupaten/Kota Tahun 2011 No.60Pelangi tampak berupa pita warna-warni berbentuk sebagian lingkaran. Urutan 4 warna pelangi yangbenar di bawah ini dari radius yang lebih kecil ke radius yang lebih besar adalah....A. merah, jingga, kuning, hijauB. biru, ungu, hijau, kuningC. hijau, biru, nila, unguD. nila, biru, hijau, kuningPembahasanUrutan warna pelangi adalah merah, jingga, kuning, hijau, biru, nila dan ungu. Cahaya berwarna merah mempunyai panjang gelombang terbesar, sedangkan cahaya berwarna ungumempunyai panjang gelombang terkecil. Pembelokan masing-masing warna cahaya bergantung padapanjang gelombang cahaya tersebut. Ketika cahaya putih matahari didispersikan oleh tetesan air hujan, cahaya yang mempunyai panjanggelombang lebih besar dibelokkan lebih sedikit sedangkan cahaya yang mempunyai panjanggelombang lebih kecil dibelokkan lebih jauh. Cahaya berwarna merah dibelokkan paling sedikit karenapanjang gelombangnya paling besar sedangkan cahaya berwarna ungu dibelokkan paling jauh karenapanjang gelombangnya paling kecil, sebagaimana ditunjukkan pada diagram di bawah. Dengandemikian mata pengamat melihat cahaya berwarna merah berada paling atas dan cahaya berwarna unguberada paling bawah. Semua warna cahaya pelangi berbentuk sebagian lingkaran, di mana lingkaran cahaya berwarna merahberada paling atas sehingga mempunyai radius (jari-jari) paling besar sedangkan lingkaran cahayaberwarna ungu berada paling bawah sehingga mempunyai radius (jari-jari) paling kecil.Urutan warna cahaya yang mempunyai radius paling besar ke radius paling kecil adalah merah, jingga,kuning, hijau, biru, nila, ungu.Jawaban yang benar adalah D.

11. Soal Olimpiade Sains Nasional SMP Tingkat Kabupaten/Kota Tahun 2011 No.35Warna-warna cahaya dari lapisan minyak di permukaan air terjadi karena peristiwa....A. interferensiB. polarisasiC. dispersiD. difraksiPembahasanJawaban yang benar adalah C.

© 2015 Gurumuda.net | A. San Lohat, S.Pd. 46

Page 49: osn fisika smp

Pembahasan Soal OSN Fisika SMP Tingkat Kabupaten/Kota

Pemuaian

1. Soal Olimpiade Sains Nasional SMP Tingkat Kabupaten/Kota Tahun 2010 No.39Cairan pengisi thermometer terbuat dari raksa atau alkohol, dasar pembuatan thermometer tersebutmemakai azas perubahan....A. angka muai raksa sebagai akibat perubahan suhu B. daya serap panas akibat perubahan suhu raksaC. tekanan sebagai akibat adanya perubahan suhu pada raksaD. volume sebagai akibat adanya perubahan suhu pada raksaPembahasanVolume benda berubah ketika suhunya berubah. Pada umumnya jika suhu meningkat maka volumebertambah, sebaliknya bila suhu menurun maka volume berkurang. Hal ini dialami juga oleh raksa ataualkohol. Jawaban yang benar adalah D.

2. Soal Olimpiade Sains Nasional SMP Tingkat Kabupaten/Kota Tahun 2010 No.42Perhatikan gambar sebuah bimetal pada suhu 26oC berikut!

Ketika bimetal tersebut dinaikkan suhunya menjadi 50oC, peristiwa yang terjadi sebagai berikut :No Bimetal membengkok Karena koefisien muai panjang logam (a)1 Ke arah logam (a) Lebih besar dari koefisien muai panjang logam (b)2 Ke arah logam (b) Lebih besar dari koefisien muai panjang logam (b)3 Ke arah logam (a) Lebih kecil dari koefisien muai panjang logam (b)4 Ke arah logam (b) Lebih kecil dari koefisien muai panjang logam (b)

Arah membengkok dan alasan yang benar ditunjukkan oleh....A. (1) dan (2)B. (1) dan (4)C. (2) dan (3)D. (3) dan (4)PembahasanBimetal merupakan gabungan dari dua keping logam yang mempunyai koefisien muai panjangberbeda, di mana salah satu keping logam mempunyai koefisien muai panjang (lebih besar sedangkankeping logam lainnya mempunyai koefisien muai panjang lebih kecil.

Apabila suhu logam bertambah maka logam mengalami pemuaian panjang (panjang logam bertambah),sebaliknya jika suhu logam berkurang maka logam mengalami penyusutan panjang (panjang logamberkurang). Untuk peningkatan suhu yang sama, kedua keping logam memuai dengan pertambahanpanjang yang berbeda. Logam yang mempunyai koefisien muai panjang lebih besar mengalamipertambahan panjang lebih besar, sedangkan logam yang koefisien muai panjang lebih kecil mengalamipertambahan panjang lebih kecil. Demikian juga sebaliknya, untuk penurunan suhu yang sama, keduakeping logam menyusut dengan pengurangan panjang yang berbeda. Logam yang mempunyaikoefisien muai panjang lebih besar mengalami penyusutan panjang lebih besar, sedangkan logam yangmempunyai koefisien muai panjang lebih kecil mengalami penyusutan panjang lebih sedikit.

Dengan demikian ketika suhu bimetal bertambah, keping logam yang mempunyai koefisien muaipanjang lebih besar membengkok ke arah logam yang mempunyai koefisien muai panjang lebih kecil.Mengapa demikian ? Karena kondisi ini memungkinkan keping logam yang mempunyai koefisien muai

© 2015 Gurumuda.net | A. San Lohat, S.Pd. 47

Page 50: osn fisika smp

Pembahasan Soal OSN Fisika SMP Tingkat Kabupaten/Kota

panjang lebih besar, menjadi lebih panjang daripada keping logam yang mempunyai koefisien muaipanjang lebih kecil. Sebaliknya ketika suhu bimetal berkurang, keping logam yang mempunyai koefisien muai panjanglebih kecil membengkok ke arah logam yang mempunyai koefisien muai panjang lebih besar. Kondisiini memungkinkan keping logam yang mempunyai koefisien muai panjang lebih besar, menjadi lebihpendek daripada keping logam yang mempunyai koefisien muai panjang lebih kecil. Jawaban yang benar adalah C.

3. Soal Olimpiade Sains Nasional SMP Tingkat Kabupaten/Kota Tahun 2010 No.45Dua buah batang mula-mula masing-masing panjangnya 10 m. Kedua batang tersebut terbuat dari bajadan aluminium dengan koefisien muai panjang berturut-turut 11 x 10-6 /oC dan 24 x 10-6 /oC. Setelahkenaikan temperatur sebesar 40oC panjang batang baja dan aluminium beturut-turut menjadi....A. 10,044 m dan 10,096 mB. 10,0044 m dan 10,0096 mC. 10,096 m dan 10,044 mD. 10,0096 m dan 10,0044 mPembahasanDiketahui :Panjang awal (Lo) batang baja = 10 meterPanjang awal (Lo) batang aluminium = 10 meterKoefisien muai panjang baja (α) = 11 x 10-6 /oCKoefisien muai panjang aluminium (α) = 24 x 10-6

/oC Perubahan suhu (ΔT) = 40oCDitanya : Panjang batang baja dan aluminiumsetelah suhunya bertambahJawab :Rumus pemuaian panjang :L = Lo + ΔLL = Lo + α Lo ΔT

Panjang batang baja setelah suhunya bertambah :L = Lo + α Lo ΔTL = 10 + (11 x 10-6)(10)(40) L = 10 + (11 x 10-6)(400) L = 10 + (4400 x 10-6) L = 10 + 0,0044L = 10,0044 meter

Panjang batang aluminium setelah suhunyabertambah :L = Lo + α Lo ΔT) L = 10 + (24 x 10-6)(10)(40) L = 10 + (24 x 10-6)(400) L = 10 + (9600 x 10-6) L = 10 + 0,0096L = 10,0096 meter Jawaban yang benar adalah B.

4. Soal Olimpiade Sains Nasional SMP Tingkat Kabupaten/Kota Tahun 2011 No.43Sebatang logam dengan koefisien muai panjang α mempunyai panjang l pada suhu 25oC. Jikadipanaskan menjadi 30oC, ia mengalami pertambahan panjang Δl. Jika logam tersebut dipotongmenjadi 1/3 panjang semula, maka pertambahan panjangnya ketika dipanaskan sampai suhunyamenjadi 35oC adalah....A. 2/3 Δl B. 3/2 ΔlC. 5/3 ΔlD. 5/2 ΔlPembahasanDiketahui :JikaKoefisien muai panjang logam (α) = αPanjang logam mula-mula (Lo) = lPertambahan suhu (ΔT) = 30oC – 25oC = 5oC makaPertambahan panjang (ΔL) = Δl Ditanya : Jika lo = 1/3 l dan ΔT = 35oC – 25oC

Hitung koefisien muai panjang logam (α)menggunakan rumus pertambahan panjang logam :ΔL = α Lo ΔT Δl = (α)(l)(5)

α=Δl5 l

Gunakan nilai koefisien muai panjang ini untukmenghitung pertambahan panjang logam ketikasuhunya berubah dari 25oC menjadi 35oC.

Pertambahan panjang=α Lo ΔT

Pertambahan panjang=Δl5 l (

13

l) (10 )

© 2015 Gurumuda.net | A. San Lohat, S.Pd. 48

Page 51: osn fisika smp

Pembahasan Soal OSN Fisika SMP Tingkat Kabupaten/Kota

= 10oC maka pertambahan panjang = ..... ? Jawab :

Pertambahan panjang=Δl15

(10 )

Pertambahan panjang=Δl3

(2 )=23

Δl

Jawaban yang benar adalah A.

5. Soal Olimpiade Sains Nasional SMP Tingkat Kabupaten/Kota Tahun 2011 No.45Pada temperatur 20oC, panjang sebuah batang aluminium 200 cm. Jika dipanaskan sampai temperatur100oC panjangnya bertambah 0,00384 m. Jika pada temperatur 20oC terdapat sebuah bola terbuat daribahan aluminium yang sama, maka agar volume bola itu bertambah 1 % dari volume bola semula, bolatersebut harus dipanaskan hingga temperatur....A. 139oCB. 159oCC. 208oCD. 228oCPembahasanDiketahui :Panjang awal batang aluminium (Lo) = 200 cm = 2 meterPerubahan suhu (ΔT) = 100oC – 20oC = 80oCPertambahan panjang (Δl) = 0,00384 meter Ditanya : Agar volume bola itu bertambah 1 % dari volume bola semula, bola tersebut harusdipanaskan hingga temperatur....Jawab :Hitung koefisien panjang batang aluminium menggunakan rumus pertambahan panjang :ΔL = α Lo ΔT

α=ΔL

Lo ΔT

α=0,00384(2 ) (80 )

α=0,00384

160α=0,000024

Koefisien muai volume aluminium (β) = 3 x koefisien muai panjang (α) = 3 (0,000024) = 0,000072

Diketahui :Volume bola semula = VPertambahan volume bola aluminium (ΔV) = (1 %)(V) = (1/100)(V) = V/100Pertambahan suhu (ΔT) = T - 20 Ditanya : Bola dipanaskan hingga suhu (T) = ..... ?Jawab :

∆ V =β V o ∆ T

V100

= (0,000072 ) (V ) (T − 20 )

VV

= (0,000072 ) (100 ) (T − 20 )

1=(0,0072 ) (T − 20 )

1=0,0072 T − 0,1441+0,144=0,0072T1,144=0,0072T

© 2015 Gurumuda.net | A. San Lohat, S.Pd. 49

Page 52: osn fisika smp

Pembahasan Soal OSN Fisika SMP Tingkat Kabupaten/Kota

T=1,1440,0072

=158,8=159

Jawaban yang benar adalah B.

6. Soal Olimpiade Sains Nasional SMP Tingkat Kabupaten/Kota Tahun 2011 No.47Sebuah plat logam berbentuk lingkaran memiliki lubang lingkaran konsentris (sepusat) di tengah.Apabila plat tersebut dipanaskan maka.....A. diameter plat membesar dan diameter lubang mengecilB. diameter plat mengecil dan diameter lubang membesarC. diameter plat membesar dan diameter lubang membesarD. diameter plat mengecil dan diameter lubang mengecilPembahasanRumus luas lingkaran (A) :

A=π r2

Rumus diameter lingkaran (D) :D = 2 rJika plat logam dipanaskan maka luas (A) plat logam bertambah. Apabila luas plat logam bertambahmaka diameter (D) atau jari-jari (r) plat logam membesar. Jika diameter (D) atau jari-jari (r) plat logammembesar maka diameter dan jari-jari lubang bertambah. Jawaban yang benar adalah C.

7. Soal Olimpiade Sains Nasional SMP Tingkat Kabupaten/Kota Tahun 2011 No.52Sepasang kerangka kacamata terbuat dari bahan plastik yang memiliki koefisien muai panjang 1,3 x 10-

4 (oC)-1. Lubang tempat lensanya berbentuk lingkaran dengan jari-jari 2,2 cm. Agar lubang-lubangtempat lensa itu dapat dipasangi lensa berjari 2,21 cm, kerangka kacamata tersebut harus dipanaskansampai mengalami perubahan suhu sebesar....A. 25oCB. 35oCC. 45oCD. 55oCPembahasanDiketahui :Koefisien muai panjang plastik (α) = 1,3 x 10-4 (oC-1)Jari-jari lubang tempat lensa (r1) = jari-jari lubang lensa mula-mula = 2,2 cmJari-jari lubang lensa setelah dipanaskan (r2) = 2,21 cmDitanya : Perubahan suhu kerangka kacamataJawab :Agar lubang tempat lensa dapat dipasangi lensa berjari-jari 2,21 cm maka setelah dipanaskan, jari-jarilensa harus bertambah 2,21 cm – 2,20 cm = 0,01 cm.Koefisien muai luas plat logam (β) = 2 x koefisien muai panjang (α) = 2 x 1,3 x 10-4 oC-1 = 2,6 x 10-4 oC-

1 Luas lingkaran mula-mula :

A1=π r12=π (2,2 )2=4,84 π

Luas lingkaran setelah dipanaskan :

A2=π r12=π (2,21 )2=4,8841 π

Pertambahan luas lingkaran :∆ A=A2− A1=4,8841 π − 4,84 π=0,0441 π

Perubahan suhu :

© 2015 Gurumuda.net | A. San Lohat, S.Pd. 50

Page 53: osn fisika smp

Pembahasan Soal OSN Fisika SMP Tingkat Kabupaten/Kota

∆ A=β A1 ∆ T

0,0441 π=(2,6 x10− 4 ) (4,84 π )∆ T

0,0441 π=12,584 π x 10− 4 ∆ T

0,0441=12,584 x 10− 4 ∆ T

∆ T=0,0441

12,584 x10− 4

∆ T=0,044112,584

x 104

∆ T=0,0035 x104

∆ T=35Jawaban yang benar adalah B.

8. Soal Olimpiade Sains Nasional SMP Tingkat Kabupaten/Kota Tahun 2011 No.49Pada gambar berikut ini tampak adanya celah sambungan antara dua rel kereta api yang terbuat daribesi. Celah itu dimaksudkan agar....

A. Gaya gesekan ketika dilalui kereta api semakin kecilB. Getaran ketika dilewati kereta api semakin kecilC. rel kereta api tidak melengkung pada saat besi memuaiD. rel kereta api tidak melengkung pada saat dilewati kereta apiPembahasanRel terbuat dari besi yang mudah memuai ketika suhunya berubah. Pada siang hari yang panas, suhu relbertambah sehingga rel memuai. Ketika bergesekan dengan roda kereta api, suhu rel juga bertambahsehingga rel bisa memuai. Karenanya fungsi celah pada sambungan antara dua rel kereta api adalahagar rel kereta api tidak melengkung pada saat besi memuai. Jawaban yang benar adalah C.

9. Soal Olimpiade Sains Nasional SMP Tingkat Kabupaten/Kota Tahun 2011 No.50Sebuah seterika listrik dilengkapi dengan termostatsehingga secara otomatis arus terputus ketikaseterika mencapai suhu tertentu. Kenaikan suhuseterika, membuat posisi bimetal melengkungseperti pada gambar. Hal ini terjadi karena....A. kalor jenis logam (B) lebih besar daripada kalorjenis logam (A)B. kalor jenis logam (B) lebih kecil daripada kalor

jenis logam (A)C. koefisien muai panjang logam (B) lebih besar dari koefisien muai panjang (A)D. koefisien muai panjang logam (B) lebih kecil daripada koefisien muai panjang (A)PembahasanBimetal terdiri dari logam A dan logam B yang mempunyai koefisien muai panjang berbeda.Berdasarkan gambar di atas, ketika suhu meningkat, tampak logam A melengkung ke arah logam B.Dengan demikian dapat disimpulkan bahwa untuk peningkatan suhu yang sama, logam A mengalamipemuaian panjang lebih banyak sedangkan logam B mempunyai pemuaian panjang lebih sedikit.Pertambahan panjang logam A lebih besar dari logam B karenanya logam A mempunyai koefisien muaipanjang lebih besar dari logam B.Jawaban yang benar adalah D.

© 2015 Gurumuda.net | A. San Lohat, S.Pd. 51

Page 54: osn fisika smp

Pembahasan Soal OSN Fisika SMP Tingkat Kabupaten/Kota

Skala Suhu, Kalibrasi termometer

Skala suhu1. Soal Olimpiade Sains Nasional SMP Tingkat Kabupaten/Kota Tahun 2010 No.40Suhu air hangat menurut skala termometer Celcius 45. Menurut skala Fahrenheit suhu tersebutadalah....A. 25B. 57C. 81D. 113PembahasanDiketahui :TC = 45o CDitanya : TF = ..... oF ?Jawab :

T F=95

T c+32

T F=95

(45 )+32

T F=9 (9 )+32

T F=81+32

TF = 113 oFJawaban yang benar adalah D.

2. Soal Olimpiade Sains Nasional SMP Tingkat Kabupaten/Kota Tahun 2011 No.51Sebuah termometer yang tidak umum menggunakan skala pengukuran dengan titik beku air -15oS dantitik didih air +60oS. Jika termometer tersebut menunjukkan nilai 10oS, nilai suhu tersebut dalam skalaFahrenheit adalah....A. 28o

B. 35o

C. 57o

D. 92o

PembahasanDiketahui :Titik beku air = -15oSTitik didih air = +60oSDitanya : 10oS = .....oFJawab :Pada skala Fahrenheit, titik beku air adalah 32oF dan titik didih air adalah 212oF. Antara titik beku dantitik didih air terdapat selisih 212o – 32o = 180o. Pada skala Celcius titik beku air adalah 0oC dan titik didih air adalah 100oC. Antara titik beku dan titikdidih air terdapat selisih 100o – 0o = 100o. Pada skala S, titik beku air adalah -15oS dan titik didih air adalah 60oS. Antara titik beku dan titik didihair terdapat selisih 60o - (-15o) = 60o + 15o = 75o.

Ubah skala S ke skala Celcius :

TC=10075

(T S+15 )

TC=43

(T S+15 )

TC=43

(10+15 )

TC=43

(25 )

TC=1003

Ubah skala Celcius ke Fahrenheit :

© 2015 Gurumuda.net | A. San Lohat, S.Pd. 52

Page 55: osn fisika smp

Pembahasan Soal OSN Fisika SMP Tingkat Kabupaten/Kota

TC=( 180100

T C)+32

TC=( 180100

1003 )+32

TC=1803

+32

TC=60+32

TC=92

Jawaban yang benar adalah D.

Kalibrasi termometer3. Soal Olimpiade Sains Nasional SMP Tingkat Kabupaten/Kota Tahun 2010 No.47Sebuah pipa gelas berisi alkohol akan digunakan sebagai termometer. Tinggi kolom alkohol ketikaujung pipa dikontakkan dengan es meleleh dan air mendidih berturut-turut adalah 4 cm dan 28 cm.Temperatur air yang membuat tinggi kolom alkohol 16 cm adalah..... oCA. 67,7B. 50C. 16D. 12PembahasanDiketahui :Tinggi kolom alkohol ketika ujung pipa dikontakkan dengan es meleleh = 4 cmTinggi kolom alkohol ketika ujung pipa dikontakkan dengan air mendidih = 28 cmDitanya : Temperatur air yang membuat tinggi kolom alkohol 16 cm adalah..... oCJawab :Suhu es meleleh (es sedang mencair) adalah 0oC sehingga tinggi kolom alkohol 4 cm setara dengan4oC. Suhu air mendidih adalah 100oC sehingga tinggi kolom alkohol 28 cm setara dengan 100oC.

Selisih tinggi kolom alkohol adalah 28 cm – 4 cm = 24 cm.Selisih skala suhu Celcius adalah 100oC – 0oC = 100oC.

100oC / 24 cm = 4,17 oC / cm Jadi kolom alkohol setinggi 1 cm setara dengan 4,17 oC.

Ketika tinggi kolom alkohol 4 cm, temperatur air adalah 0oC. Jika tinggi kolom alkohol 16 cm, berapasuhu air dalam skala Celcius ? 16 cm – 4 cm = 12 cm. Jadi 12 x 4,17 = 50,04 oC. Jawaban yang benar adalah B.

4. Soal Olimpiade Sains Nasional SMP Tingkat Kabupaten/Kota Tahun 2011 No.41Yang tidak diperlukan untuk menentukan skala termometer adalah....A. titik tetap atasB. titik didihC. titik tetap bawahD. lebar skalaPembahasan

© 2015 Gurumuda.net | A. San Lohat, S.Pd. 53

Page 56: osn fisika smp

Pembahasan Soal OSN Fisika SMP Tingkat Kabupaten/Kota

Yang diperlukan untuk menentukan skala termometer adalah titik tetap atas (titik didih) dan titik tetapbawah (titik beku). Jadi yang tidak diperlukan untuk menentukan skala termometer berdasarkan soal diatas adalah lebar skala.Jawaban yang benar adalah D.

5. Soal Olimpiade Sains Nasional SMP Tingkat Kabupaten/Kota Tahun 2011 No.42Sebuah termometer tak berskala berisi air raksa. Ketika dimasukkan ke dalam es melebur panjangkolom raksa 5 cm dan ketika dimasukkan ke dalam air mendidih panjang kolom raksa 30 cm. Ketikatermometer menunjukkan 25oC panjang kolom raksa adalah....A. 6,25 cm B. 10,25 cmC. 11,25 cmD. 22,50 cmPembahasanDiketahui :Panjang kolom raksa ketika dimasukkan ke dalam es melebur = 5 cmPanjang kolom raksa ketika dimasukkan ke dalam air mendidih = 30 cmDitanya : Panjang kolom raksa ketika termometer menunjukkan 25oCJawab :Suhu es melebur (es sedang mencair) adalah 0oC karenanya panjang kolom raksa 5 cm setara dengan0oC. Suhu air mendidih adalah 100oC karenanya panjang kolom raksa 30 cm setara dengan 100oC.

Selisih panjang kolom raksa adalah 30 cm – 5 cm = 25 cm.Selisih skala suhu Celcius adalah 100oC – 0oC = 100oC.

25 cm / 100oC = 0,25 cm / oC Apabila kolom raksa bertambah 0,25 cm maka setara dengan penambahan suhu air sebesar 1 oC.

Ketika suhu air 0oC, panjang kolom raksa adalah 5 cm. Jadi jika suhu air 25oC maka panjang kolomraksa adalah 5 cm + (0,25 x 25) = 5 cm + 6,25 cm = 11,25 cm. Jawaban yang benar adalah C.

© 2015 Gurumuda.net | A. San Lohat, S.Pd. 54

Page 57: osn fisika smp

Pembahasan Soal OSN Fisika SMP Tingkat Kabupaten/Kota

Kalor, Perubahan Wujud, Perpindahan Kalor, Asas Black

Kalor lebur1. Soal Olimpiade Sains Nasional SMP Tingkat Kabupaten/Kota Tahun 2010 No.43Kalor lebur air adalah 80 kal/gram. Ini berarti....A. dibutuhkan 80 kalori untuk menaikkan temperatur 1 gram air sebesar 1 derajatB. dibutuhkan 80 kalori untuk mengubah 1 gram air menjadi uap seluruhnyaC. dibutuhkan 80 kalori untuk mempertahankan temperatur 1 gram airD. dibutuhkan 80 kalori untuk mengubah 1 gram es menjadi air seluruhnyaPembahasanJawaban yang benar adalah D.

Perubahan wujud2. Soal Olimpiade Sains Nasional SMP Tingkat Kabupaten/Kota Tahun 2010 No.41Perhatikan grafik hasil suatu percobaan berikut!

Air sebanyak 2 kg bersuhu 60oC akan diuapkan. Jika kaloruap air 2,27 x 106 J/kg, kalor jenis air 4200 J/(kg oC) dantekanan udara 76 cmHg, maka kalor yang diperlukan untukproses B-C adalah....A. 336 kilojouleB. 4540 kilojouleC. 4876 kilojouleD. 5212 kilojoule

PembahasanDiketahui :Massa air (m) = 2 kgSuhu air (T) = 60oC Kalor uap air (LV) = 2,27 x 106 J/kgKalor jenis air (c) = 4200 J/(kg oC) Tekanan udara (P) = 76 cmHg Ditanya : Kalor yang diperlukan untuk proses B-CJawab :Proses A-B = Air dipanaskan dari suhu 60oC sampai 100oC. Pada proses ini, kalor digunakan untukmenaikan suhu air. Kalor dihitung menggunakan rumus : Q = m c ΔTProses B-C = Air diuapkan. Pada proses ini, kalor tidak digunakan untuk menaikkan suhu tetapidigunakan untuk menguapkan air. Kalor dihitung menggunakan rumus : Q = m LV Proses C- = Uap dipanaskan. Pada proses ini, kalor digunakan untuk menaikan suhu uap. Kalordihitung menggunakan rumus : Q = m c ΔT.

Jadi jika suhu air berubah maka rumus kalor yang digunakan adalah Q = m c ΔT, apabila wujud airberubah maka rumus kalor yang digunakan adalah Q = m LV.

Kalor yang diperlukan untuk proses B-C :Q = m LV

Q = (2 kg)(2,27 x 106 J/kg)Q = 4,54 x 106 JouleQ = 4.540.000 JouleQ = 4.540 kilojoule

© 2015 Gurumuda.net | A. San Lohat, S.Pd. 55

Page 58: osn fisika smp

Pembahasan Soal OSN Fisika SMP Tingkat Kabupaten/Kota

Jawaban yang benar adalah B.

3. Soal Olimpiade Sains Nasional SMP Tingkat Kabupaten/Kota Tahun 2011 No.44Berikut ini adalah kelompok perubahan wujud yang membebaskan kalor :A. Membeku, mengembun, menyublim B. Mencair, menguap, menghablurC. Mencair, menguap, mengembunD. Membeku, menguap, menyublim PembahasanSecara umum terdapat dua jenis perubahan wujud yakni perubahan wujud yang menyerap kalor danperubahan wujud yang membebaskan/melepaskan kalor. Pada perubahan wujud yang menyerap kalor, benda menyerap kalor hingga kalor pada bendabertambah lalu wujud benda berubah. Pada perubahan wujud yang melepaskan kalor, bendamelepaskan kalor hingga kalor pada benda berkurang lalu wujud benda berubah. Jika benda menyerap kalor maka wujud benda berubah dari padat ke cair ke gas.Jika benda melepaskan kalor maka wujud benda berubah dari gas ke cair ke padat.

Perhatikan jawaban soal di atas.Membeku = wujud benda berubah dari cair ke padat (melepaskan kalor)Mengembun = wujud benda berubah dari gas ke cair (melepaskan kalor)Menyublim = wujud benda berubah dari gas ke padat (melepaskan kalor)Mencair = wujud benda berubah dari padat ke cair (menyerap kalor)Menguap = wujud benda berubah dari cair ke gas (menyerap kalor)Menghablur = wujud benda berubah dari padat ke gas (menyerap kalor)Jawaban yang benar adalah A. Perpindahan Kalor4. Soal Olimpiade Sains Nasional SMP Tingkat Kabupaten/Kota Tahun 2010 No.46Sebuah jendela kaca mempunyai tebal 8 mm dan luas 1 m2. Jika temperatur udara di luar 30oC dan didalam 20oC serta konduktivitas termal kaca adalah 0,8 W/moC, maka jumlah energi yang melewati kacaselama 1 jam adalah.... mega jouleA. 360 B. 36C. 6D. 3,6PembahasanDiketahui :Tebal kaca (l) = 8 mm = 8 / 1000 = 0,008 mLuas kaca (A) = 1 m2 Perbedaan suhu (ΔT) = 30oC–20oC = 10oCKonduktivitas termal kaca (k) = 0,8 W/moCSelang waktu (t) = 1 jam = 60 (60 sekon) = 3600 sekon Ditanya : Jumlah energi yang melewati kaca selama 1 jamJawab :Rumus laju perpindahan kalor secara konduksi :

Qt=k A

ΔTl

Jumlah energi yang melewati kaca selama 1 jam :

© 2015 Gurumuda.net | A. San Lohat, S.Pd. 56

Page 59: osn fisika smp

Pembahasan Soal OSN Fisika SMP Tingkat Kabupaten/Kota

Q3600

= (0,8 ) (1 )(10 )

0,008Q

3600= (100 ) (1 ) (10 )

Q3600

=1000

Q=(1000 ) (3600 )

Q=3.600 .000 JouleQ=3,6 megaJoule

Jawaban yang benar adalah D.

5. Soal Olimpiade Sains Nasional SMP Tingkat Kabupaten/Kota Tahun 2011 No.46Sebuah batang logam panjangnya 1 m. Salah satu ujungnya dipanaskan pada suhu tetap 150oCsementara ujung yang lain dipertahankan pada suhu ruang yaitu 30oC. Temperatur bagian batang yangberjarak 30 cm dari ujung yang dipanaskan adalah....A. 36oCB. 84oCC. 114oCD. 125oCPembahasanDiketahui :Panjang batang logam (l) = 1 meterPerbedaan suhu (ΔT) = 150oC–30oC = 120oC Ditanya : Temperatur bagian batang yang berjarak 30 cm dari ujung yang dipanaskanJawab :Hitung laju perpindahan kalor secara konduksi :

Qt=kA

ΔTl

Qt=kA

1201

Qt=kA 120

Hitung temperatur bagian batang yang berjarak 30 cm dari ujung yang dipanaskan :Panjang batang logam (l) = 30 cm = 0,3 meterPerbedaan suhu (ΔT) = 150–T

Qt=kA

ΔTl

kA 120=kA150−T

0,3

120=150 −T

0,3(0,3 ) (120 )=150−T36=150− TT=150 −36T=114 oC

Jawaban yang benar adalah C.

© 2015 Gurumuda.net | A. San Lohat, S.Pd. 57

Page 60: osn fisika smp

Pembahasan Soal OSN Fisika SMP Tingkat Kabupaten/Kota

Asas Black6. Soal Olimpiade Sains Nasional SMP Tingkat Kabupaten/Kota Tahun 2010 No.44Berapa suhu akhir campuran jika 50 gram air bersuhu 0oC ditambahkan ke 100 gram air bersuhu 90oC ?A. 15oCB. 45oCC. 60oCD. 75oCPembahasanDiketahui :Massa air 1 (m1) = 50 gram Suhu air 1 (T1) = 0oCMassa air 2 (m2) = 100 gram Suhu air 2 (T2) = 90oCDitanya : Berapa suhu akhir campuran ?Jawab :Asas Black menyatakan bahwa kalor yang dilepas air bersuhu tinggi sama dengan kalor yang diserapair bersuhu rendah.Q lepas = m c ΔT = (50)(c air)(T–0) = (50)(c air)(T)Q serap = m c ΔT = (100)(c air)(90–T) = (100)(c air)(90-T)

Q lepas = Q serap(50)(c air)(T) = (100)(c air)(90-T)50 T = 100 (90-T)50 T = 9000 – 100 T50 T + 100 T = 9000 150 T = 9000T = 9000 / 150 = 900 / 15T = 60oC Jawaban yang benar adalah C.

7. Soal Olimpiade Sains Nasional SMP Tingkat Kabupaten/Kota Tahun 2011 No.40Pada temperatur ruangan 0oC dan tekanan udara 1 atm, 10 gram es dimasukkan ke dalam 10 gram air didalam suatu wadah. Jika temperatur ruangan dan tekanan udara dijaga tetap, maka pada akhirnya yangdidapatkan di dalam wadah adalah...A. 10 gram es dan 10 gram airB. 20 gram esC. 20 gram airD. 5 gram es dan 15 gram airPembahasanEs berwujud padat dicampur dengan air berwujud cair pada suhu 0oC dan suhu dijaga agar selalu 0oC.Bila suhu tetap 0oC maka tidak ada perpindahan kalor di antara es dan air. Jadi pada akhirnya tetap ada10 gram es dan 10 gram air.Jawaban yang benar adalah A.

Catatan :- Kata ES menerangkan bahwa air berwujud padat dan kata AIR menerangkan bahwa air berwujud cair.- Jika suhu es kurang dari 0oC maka sebelum mencair, suhu es harus naik hingga 0oC

© 2015 Gurumuda.net | A. San Lohat, S.Pd. 58

Page 61: osn fisika smp

Pembahasan Soal OSN Fisika SMP Tingkat Kabupaten/Kota

- Pada suhu 0oC terjadi perubahan wujud air dari padat (es) menjadi cair (air). Jadi es bisa bersuhu 0oCdan air juga bisa bersuhu 0oC. Ketika bersuhu 0oC, es berwujud padat sedangkan air berwujud cair.

© 2015 Gurumuda.net | A. San Lohat, S.Pd. 59

Page 62: osn fisika smp

Pembahasan Soal OSN Fisika SMP Tingkat Kabupaten/Kota

Muatan listrik, Gaya listrik

Muatan listrik1. Soal Olimpiade Sains Nasional SMP Tingkat Kabupaten/Kota Tahun 2011 No.57Dua buah muatan identik masing-masing dengan massa m = 9√3 x 10-4 kg digantung pada satu titikyang sama menggunakan tali yang panjangnya l = 1 meter. Jika kedua muatan tersebut terpisah sejauh a= 1 meter maka besar masing-masing muatan adalah....A. 4 mikrocoulombB. 2 mikrocoulombC. 1 mikrocoulombD. 0,5 mikrocoulombPembahasanDiketahui :Dua buah muatan identik artinya dua buah muatan sejenis, di mana muatan sejenis saling tolakmenolak.Panjang tali (l) = 1 meterJarak antara kedua muatan (r) = 1 meter Massa muatan (m) = 9√3 x 10-4 kgPercepatan gravitasi (g) = 10 m/s2

Gaya berat (w) = m g = (9√3 x 10-4 kg)(10 m/s2) = 9√3 x 10-3 Newton Ditanya : Besar masing-masing muatan (q)Jawab :

Keterangan gambar :F = Gaya listrik tolak menolak antara beban 1 dan 2w1 = Gaya berat bola 1w2 = Gaya berat bola 2T1 = Gaya tegangan tali 1T1x = Komponen gaya tegangan tali 1 pada sumbu xT1y = Komponen gaya tegangan tali 1 pada sumbu y T2 = Gaya tegangan tali 2T2x = Komponen gaya tegangan tali 2 pada sumbu xT2y = Komponen gaya tegangan tali 2 pada sumbu y

l1 = panjang tali 1l1x = komponen panjang tali pada sumbu xl1y = komponen panjang tali pada sumbu y

Hitung nila i cos θ :

cosθ=l1 y

l1

l12 = l1x

2 + l1y2

l1y2 = l1

2 – l1x2 = l1

2 – l1x2 = 12 – 0,52

= 1 – 0,25 = 0,75 = (0,25) 3 l1y

= √(0,25) 3 = 0,5√3 meter

cosθ=l1 y

l1

=0,5√ 3

1=0,5√ 3

Hitung gaya tegangan tali (T1) :

© 2015 Gurumuda.net | A. San Lohat, S.Pd. 60

Page 63: osn fisika smp

Pembahasan Soal OSN Fisika SMP Tingkat Kabupaten/Kota

Diketahui T1y = w = 9√3 x 10-3 Newton

cosθ=T 1 y

T 1

T1=T1 y

cos θ=

9√ 3 x10− 3

0,5√ 3=18x 10−3 Newton

Hitung nila i sin θ :

sin θ=l1x

l1

sin θ=0,51

sin θ=0,5

Hitung komponen gaya tegangan tali pada sumbu x (T1x) :

sin θ=T 1 x

T 1

T1 x=T1 sin θ

T1 x=(18 x 10− 3 ) (0,5 )

T1 x=9 x10− 3

Kedua bola setimbang sehingga T1x = F = 9 x 10-3 Newton

Hitung muatan masing-masing bola menggunakan rumus hukum Coulomb :

F=kq1 q2

r2

9 x 10− 3=

(9 x 109 ) (q ) (q )

12

9 x 10− 3=

(9 x109 ) q2

1

9 x10− 3=(9 x 109 ) q

2

q2=

9 x 10−3

9 x109

q2=1 x 10−3 x10− 9

q2=1x 10−12

q=√1x 10−12

q=1 x 10− 6

Besar masing-masing muatan adalah 1 x 10-6 Coulomb atau 1 mikroCoulomb.Jawaban yang benar adalah C.

Menjadikan benda bermuatan listrik2. Soal Olimpiade Sains Nasional SMP Tingkat Kabupaten/Kota Tahun 2010 No.50Dua cara pemberian muatan listrik pada suatu benda adalah....A. Gosokan dan elektromagnetisasiB. Induksi dan elektromagnetisasiC. Gosokan dan induksiD. Induksi dan magnetisasiPembahasanDua cara pemberian muatan listrik pada suatu benda adalah gosokan dan induksi.

© 2015 Gurumuda.net | A. San Lohat, S.Pd. 61

Page 64: osn fisika smp

Pembahasan Soal OSN Fisika SMP Tingkat Kabupaten/Kota

Gosokan adalah cara menjadikan suatu benda bermuatan listrik positif atau bermuatan listrik negatifdengan menggosokkan benda tersebut dengan benda lainnya, misalnya menggosokan batang kacadengan kain wol atau menggosokan sisir plastik dengan rambut kering. Selama bergesekan, elektronberpindah dari suatu benda ke benda lainnya sehingga salah satu benda berkelebihan elektron(berkekurangan proton) lalu menjadi bermuatan negatif dan benda lainnya berkekurangan elektron(berkelebihan proton) lalu menjadi bermuatan positif.Induksi adalah cara menjadikan suatu benda bermuatan dengan mendekatkan benda bermuatan listrikdengan bola logam netral tetapi keduanya tidak saling bersentuhan. Benda yang dijadikan bermuatanlistrik harus mempunyai elektron bebas karena elektron bebas-elektron bebas tersebut dapat berpindahtempat ketika diinduksi oleh benda bermuatan listrik. Benda-benda yang terbuat dari logam padaumumnya mempunyai elektron bebas. Benda seperti kaca, plastik atau kayu tidak bisa dijadikanbermuatan listrik dengan cara induksi karena benda-benda seperti ini tidak mempunyai elektron bebas.Jawaban yang benar adalah C.

Jenis muatan listrik3. Soal Olimpiade Sains Nasional SMP Tingkat Kabupaten/Kota Tahun 2010 No.49Empat buah benda bermuatan (P, Q, R dan S) berkelakuan sebagai berikut. P dan Q saling menolak, Pdan R saling menarik, R dan S saling menolak. Pernyataan berikut ini yang benar adalah....A. jika P bermuatan negatif, maka S bermuatan negatifB. jika Q bermuatan negatif, maka S bermuatan negatifC. jika R bermuatan negatif, maka P bermuatan negatifD. jika S bermuatan negatif, maka P bermuatan positifPembahasanP dan Q saling menolak karenanya P dan Q mempunyai muatan sejenisP dan R saling menarik karenanya P dan R mempunyai muatan tak sejenisR dan S saling menolak karenanya R dan S mempunyai muatan sejenis

Jadi jika P dan Q bermuatan positif maka R dan S bermuatan negatif atau jika P dan Q bermuatannegatif maka R dan S bermuatan positif.Jawaban yang benar adalah D.

4. Soal Olimpiade Sains Nasional SMP Tingkat Kabupaten/Kota Tahun 2011 No.59Sebuah benda dikatakan bermuatan positif apabila....A. Benda menerima protonB. Benda melepaskan protonC. Benda menerima elektronD. Benda melepaskan elektronPembahasanProton bermuatan positif sedangkan elektron bermuatan negatif. Proton berada di pusat atom sehinggaproton tidak bisa berpindah dari suatu benda ke benda lainnya, sebaliknya elektron berada dipermukaan atom sehingga bisa berpindah dari suatu benda ke benda lainnya. Benda menjadi bermuatanlistrik karena benda melepaskan elektron atau benda menerima elektron. Apabila suatu bendamenerima elektron maka benda berkelebihan elektron (kekurangan proton) sehingga benda bermuatannegatif. Jika suatu benda melepaskan elektron maka benda berkekurangan elektron (kelebihan proton)sehingga benda bermuatan positif. Jadi berdasarkan soal di atas, sebuah benda dikatakan bermuatan positif apabila benda melepaskanelektron.Jawaban yang benar adalah D.

© 2015 Gurumuda.net | A. San Lohat, S.Pd. 62

Page 65: osn fisika smp

Pembahasan Soal OSN Fisika SMP Tingkat Kabupaten/Kota

Gaya listrik5. Soal Olimpiade Sains Nasional SMP Tingkat Kabupaten/Kota Tahun 2010 No.48Dua buah muatan listrik masing-masing Q1 dan Q2 berada pada jarak R memiliki gaya tolak menolaksebesar 100 N. Jika jarak antara kedua muatan diperkecil menjadi 1/3 R, maka gaya tolak-menolakmuatan listrik menjadi....A. 33,33 N B. 100 NC. 300 ND. 900 NPembahasanDiketahui :Muatan listrik 1 = Q1

Muatan listrik 2 = Q2

Jarak antara kedua muatan = RGaya tolak menolak antara kedua muatan (F) = 100 NewtonGaya tolak menolak artinya kedua muatan listrik berjenis sama, keduanya positif atau keduanyanegatif.Ditanya : Apabila jarak antara kedua muatan diperkecil menjadi 1/3 R, maka gaya tolak menolakmuatan listrik menjadi.....Jawab :Rumus hukum Coulomb :

F=kQ1 Q2

R2

Misalkan R = 1 maka nilai Q1.Q2 dapat dihitung :

100=kQ1 Q2

12

100k

=Q1 Q2

1

Q1Q2=100k

Apabila jarak antara kedua muatan diperkecilmenjadi 1/3 R, maka gaya tolak menolak muatanlistrik (F) menjadi :

QQ

k (1 Q2 ): R2

( 1Q2 )

R2 =F=k

F=k ( 100k ) :

1

32=100:

19

F=(100 )( 91 )=(100 ) (9 )

F=900 NewtonJawaban yang benar adalah D.

6. Soal Olimpiade Sains Nasional SMP Tingkat Kabupaten/Kota Tahun 2011 No.53Muatan titik Q1 = +4e dan muatan titik Q2 = -3e ditahan tetap pada jarak 0,3 m satu sama lain, sepertipada gambar di bawah ini. Agar gaya listrik yang bekerja pada muatan Q2 adalah nol, muatan titik Q3 =+16e harus diletakkan di....

A. 0,3 m di sebelah kiri Q1

B. 0,6 m di sebelah kanan Q1

C. 0,6 m di sebelah kanan Q2

D. 0,9 m di sebelah kiri Q2

PembahasanDiketahui :Muatan sebuah elektron (-e) atau proton (+e) =1,60 x 10-19 Coulomb

Agar resultan gaya listrik yang bekerja padamuatan Q2 adalah nol maka besar gaya listrikantara muatan Q2 dan muatan Q3 (F32) = besargaya listrik antara muatan Q1 dan muatan Q2 (F12)

© 2015 Gurumuda.net | A. San Lohat, S.Pd. 63

Page 66: osn fisika smp

Pembahasan Soal OSN Fisika SMP Tingkat Kabupaten/Kota

Muatan titik Q1 = +4e Muatan titik Q2 = -3e Muatan titik Q3 = +16e Jarak antara muatan titik Q1 dan muatan titik Q2 =0,3 meter Ditanya : Agar gaya listrik yang bekerja padamuatan Q2 adalah nol, muatan titik Q3 = +16eharus diletakkan di....Jawab :Gaya listrik antara muatan Q1 dan muatan Q2 :

F12=kQ1Q2

R122

F12=k(4 e ) (3e )

(0,3 )2

F12=k12e2

0,09Muatan Q1 positif dan muatan Q2 negatif sehinggaarah gaya listrik F12 ke kiri (menuju muatan Q1).

dan arah F32 harus berlawanan dengan arah F12.Arah F12 ke kiri karenanya arah F32 harus kekanan. Mengingat Q3 adalah positif dan Q2 adalahmuatan negatif, agar arah F32 ke kanan makamuatan Q3 harus diletakkan sebelah kanan Q2

Jarak antara muatan Q2 dan muatan Q3 dihitungmenggunakan rumus hukum Coulomb :

F32=kQ3Q 2

R322

F32=k(16e ) (3e )

R322

k12e2

0,09=k

48 e2

R322

120,09

=48

R322

10,09

=4

R322

R322=4 (0,09 )

R322=0,36

R32=√0,36

R32=0,6 meter

Jawaban yang benar adalah C.

© 2015 Gurumuda.net | A. San Lohat, S.Pd. 64

Page 67: osn fisika smp

Pembahasan Soal OSN Fisika SMP Tingkat Kabupaten/Kota

Rangkaian listrik, Hambatan listrik

Rangkaian listrik1. Soal Olimpiade Sains Nasional SMP Tingkat Kabupaten/Kota Tahun 2010 No.52Perhatikan gambar rangkaian listrik berikut. Besar arus listrik yang mengalir pada hambatan 8 Ohmadalah....A. 1,8 AmpereB. 1,2 AmpereC. 0,8 AmpereD. 0,6 AmperePembahasanDiketahui :Resistor 1 (R1) = 12 ΩResistor 2 (R2) = 8 ΩResistor 3 (R3) = 10 ΩTegangan listrik (V) = 12 VoltDitanya : Kuat arus melalui R1

Jawab :Arus listrik mengalir dari potensial tinggi ke potensial rendah. Arah arus listrik pada rangkaian di atassama dengan arah putaran jarum jam.

Kuat arus yang mengalir keluar dari bateraiTerlebih dahulu hitung resistor pengganti (R). Setelah itu hitung kuat arus menggunakan rumus hukumOhm : V = I R atau I = V / R di mana V = tegangan, I = kuat arus, R = resistor pengganti

Resistor pengganti :Resistor R1 dan resistor R2 tersusun seri. Resistor pengganti adalah :R12 = R1 + R2 = 12 + 8 = 20 Ω Resistor R12 dan resistor R3 tersusun paralel. Resistor pengganti adalah :1/R = 1/R12 + 1/R3 = 1/20 + 1/10 = 1/20 + 2/20 = 3/20 R = 20/3 Ω

Kuat arus yang mengalir keluar dari baterai : I = V / R = 12 : 20/3 = 12 x 3/20 = 36/20 = 1,8 AmpereKuat arus yang mengalir keluar dari baterai adalah 1,8 Ampere Ampere.

Kuat arus listrik yang melalui resistor 8 Ω

Vab = 12 VoltR12 = 20 ΩR3 = 10 Ω

Kuat arus listrik yang melalui resistor 10 Ω I3 = Vab / R3 = 12 Volt / 10 Ω = 1,2 AmpereKuat arus listrik yang melalui resistor 20 Ω I12 = Vab / R12 = 12 Volt / 20 Ω = 0,6 Ampere

© 2015 Gurumuda.net | A. San Lohat, S.Pd. 65

Page 68: osn fisika smp

Pembahasan Soal OSN Fisika SMP Tingkat Kabupaten/Kota

Hukum I Kirchhoff menyatakan bahwa kuat arus listrik yang masuk pada suatu cabang sama dengankuat arus listrik yang keluar dari cabang tersebut. Berdasarkan hukum I Kirchhoff disimpulkan bahwakuat arus listrik yang melewati resistor 20 Ω = kuat arus listrik yang melewati resistor 12 Ω = kuat aruslistrik yang melewati resistor 8 Ω = 0,6 Ampere.Jawaban yang benar adalah D.

2. Soal Olimpiade Sains Nasional SMP Tingkat Kabupaten/Kota Tahun 2010 No.51Perhatikan rangkaian listrik tertutup berikut!

Jika hambatan dalam sumber tegangan diabaikan, maka bedapotensial antara ujung-ujung penghambat 6 Ohm adalah....A. 3 VoltB. 2 VoltC. 2/3 VoltD. 1/3 VoltPembahasan

Diketahui :Resistor 1 = 4 ΩResistor 2 = 3 ΩResistor 3 = 6 Ω Tegangan (V) = 6 voltDitanya : Beda potensial antara ujung-ujung penghambat 6 OhmJawab :Resistor 3 Ω dan resistor 3 Ω tersusun paralel. Resistor pengganti adalah :1/R23 = 1/3 Ω + 1/6 Ω = 2/6 Ω + 1/6 Ω = 3/6 ΩR23 = 6/3 Ω = 2 Ω

Resistor 4 Ω dan resistor 2 Ω tersusun seri. Resistor pengganti adalah :R = 4 Ω + 2 Ω = 6 Ω

Kuat arus listrik yang keluar dari sumber tegangan :I = V / R = 6 volt / 6 ohm = 1 AmpereResistor 4 Ω dan resistor 2 Ω tersusun seri sehingga menuruthukum I Kirchooff, kuat arus listrik yang melewati resistor 4 Ωkuat arus listrik yang melewati resistor 2 Ω = 1 Ampere.

Beda potensial antara titik a dan b :V = I R = (1 Ampere)(2 Ohm) = 2 voltBeda potensial antara titik a dan b = beda potensial antara ujung-ujung resistor 2 Ω = beda potensialantara ujung-ujung resistor 3 Ω = beda potensial antara ujung-ujung resistor 6 Ω = 2 volt.Jawaban yang benar adalah B.

Hambatan listrik3. Soal Olimpiade Sains Nasional SMP Tingkat Kabupaten/Kota Tahun 2010 No.57Tiga buah hambatan listrik, jika disusun paralel nilai hambatannya 12/11 Ohm, jika disusun seribesarnya menjadi 12 Ohm, maka besar hambatan tersebut masing-masing....A. 1 Ohm, 2 Ohm, 3 OhmB. 2 Ohm, 4 Ohm, 6 OhmC. 1 Ohm, 3 Ohm, 5 OhmD. 3 Ohm, 4 Ohm, 5 Ohm

© 2015 Gurumuda.net | A. San Lohat, S.Pd. 66

Page 69: osn fisika smp

Pembahasan Soal OSN Fisika SMP Tingkat Kabupaten/Kota

PembahasanApabila hambatan listrik disusun seri maka besar hambatan pengganti dihitung menggunakan rumus :R = R1 + R2 + R3

12 = R1 + R2 + R3 Pilihan yang mungkin adalah jawaban B dan D. Jika hambatan listrik disusun paralel maka besar hambatan pengganti dihitung menggunakan rumus :1/R = 1/R1 + 1/R2 + 1/R3. Jika menggunakan jawaban B maka :1/R = 1/2 + 1/4 + 1/6 1/R = 6/12 + 3/12 + 2/121/R = 11/12R = 12/11Jawaban yang benar adalah B.

Hambatan jenis4. Soal Olimpiade Sains Nasional SMP Tingkat Kabupaten/Kota Tahun 2010 No.56Resistivitas tungsten dan timah berturut-turut adalah 5,5 x 10-8 Ohm meter dan 22 x 10-8 Ohm meter.Kawat tungsten dan kawat timah mempunyai panjang masing-masing 5 meter. Jika kedua kawatmemiliki nilai hambat sama dan diameter tungsten adalah 2 mm maka diameter kawat timah adalah....mmA. 2 B. 4C. √2D. ½ √2PembahasanDiketahui :Resistivitas tungsten (ρ1) = 5,5 x 10-8 Ohm meter

Resistivitas timah (ρ2) = 22 x 10-8 Ohm meter

Panjang kawat tungsten (L1) = 5 meterPanjang kawat timah (L2) = 5 meter Diameter tungsten (D1) = 2 mmJari-jari tungsten (R1) = 1/2 (D) = 1/2 (2 mm) = 1 mm = 1 x 10-3 meter Ditanya : Diameter timah (D2)Jawab :Luas permukaan kawat tungsten :

A1 = r2 = (10-3)2 = 10-6

Nilai hambat kawat tungsten dan kawat timah sama.R1 = R2

ρ1 L1 / A1 = ρ2 L2 / A2

(5,5 x 10-8)(5) / 10-6 = (22 x 10-8)(5) / r2

5,5 / 10-6 = 22 / r2

5,5 r2 = 22 x 10-6

r2 = (22 x 10-6 ) / 5,5r2 = 4 x 10-6

r = 2 x 10-3 meterr = 2 milimeterJawaban yang benar adalah A.

© 2015 Gurumuda.net | A. San Lohat, S.Pd. 67

Page 70: osn fisika smp

Pembahasan Soal OSN Fisika SMP Tingkat Kabupaten/Kota

Energi dan Daya listrik, Gerak Partikel Bermuatan

Energi dan daya listrik1. Soal Olimpiade Sains Nasional SMP Tingkat Kabupaten/Kota Tahun 2010 No.53Tarif energi listrik per kWh adalah Rp. 400,-. Di sebuah rumah pesawat TV 100 Watt setiap haribekerja rata-rata 4 jam dan 2 lampu masing-masing 20 Watt digunakan 10 jam/hari. Biaya pemakaianenergi listrik per bulan (30 hari) sebesar....A. Rp. 4.800,-B. Rp. 9.600,-C. Rp. 19.200,-D. Rp. 21.120,-PembahasanDiketahui :Tarif energi listrik adalah Rp. 400/kWh Daya TV = 1 x 100 Watt = 100 Watt dan waktu pemakaian TV = 4 jam x 30 = 120 jamDaya lampu = 2 x 20 Watt = 40 Watt dan waktu pemakaian lampu = 10 jam x 30 = 300 jamDitanya : Biaya pemakaian energi listrik per bulanJawab :Energi listrik = daya listrik x waktu pemakaianEnergi listrik TV = 100 Watt x 120 jam = 12.000 Watt jamEnergi lampu = 40 Watt x 300 jam = 12.000 Watt jam

Energi total = 12.000 Watt jam + 12.000 Watt jam = 24.000 Watt jam = 24 kilo Watt jam = 24 kilo Watthour = 24 kWhBiaya pemakaian energi listrik per bulan = (24 kWh)(Rp. 400/kWh) = (24)(Rp. 400) = Rp. 9.600,-Jawaban yang benar adalah B.

2. Soal Olimpiade Sains Nasional SMP Tingkat Kabupaten/Kota Tahun 2011 No.55Dua buah lampu masing-masing bertuliskan 24 Watt / 12 Volt dihubungkan paralel, lalu dihubungkansecara seri dengan resistor 2 Ohm. Rangkaian ini kemudian dihubungkan dengan sumber tegangansehingga kedua lampu menyala dengan tingkat terang hanya 25% dari terang maksimumnya. Jika salahsatu lampu tiba-tiba putus, maka lampu yang lain menyala dengan daya....A. 12 WattB. 7,5 WattC. 6 WattD. 3 WattPembahasanRumus daya :P = V I I = P VatauP = V I -----> I = V / RP = V (V / R) P = V2 / RR = V2 / PatauP = V I -----> V = I RP = (I R) I

© 2015 Gurumuda.net | A. San Lohat, S.Pd. 68

Page 71: osn fisika smp

Pembahasan Soal OSN Fisika SMP Tingkat Kabupaten/Kota

P = I2 R I2 = P / RKeterangan : P = daya listrik, V = tegangan listrik, I = kuat arus listrik, R = hambatan lampu

Hitung hambatan rangkaian :Dua buah lampu masing-masing bertuliskan 24 Watt / 12 Volt, artinya masing-masing lampu menyalaterang maksimum dengan daya 24 Watt jika terpasang pada tegangan 12 Volt. Hambatan masing-masing lampu dihitung menggunakan rumus daya :R = V2 / P = 122 / 24 = 144 / 24 = 6 Ohm Kedua lampu terangkai paralel sehingga hambatan penggantinya adalah :1/R12 = 1/R1 + 1/R2 = 1/6 + 1/6 = 2/6 R12 = 6/2 = 3 OhmKedua lampu terangkai seri dengan resistor 2 Ohm sehingga hambatan penggantinya adalah :R = 3 Ohm + 2 Ohm = 5 OhmJadi hambatan rangkaian = R = 5 Ohm.

Hitung kuat arus listrik yang mengalir pada rangkaian :Jika kedua lampu menyala dengan tingkat terang 25% maka kedua lampu menyala dengan daya 25/100x 24 Watt = 1/4 x 24 = 24/4 = 6 Watt. Kuat arus listrik yang mengalir pada lampu dihitung menggunakan rumus daya :I2 = P / R = 6 Watt / 6 Ohm = 1 Ampere Kuat arus listrik yang mengalir pada masing-masing lampu adalah 1 Ampere. Kedua lampu terangkaiparalel karenanya menurut hukum I Kirchhoff, kuat arus listrik total yang mengalir pada kedua lampuadalah 2 Ampere. Kedua lampu dihubungkan secara seri dengan resistor 2 Ohm sehingga kuat aruslistrik yang mengalir pada resistor 2 Ohm juga bernilai 2 Ampere. Jadi kuat arus listrik yang mengalir pada rangkaian = I = 2 Ampere.

Hitung tegangan listrik sumber ggl :Tegangan listrik adalah :V = I R = (2 Ampere)(5 Ohm) = 10 VoltIni adalah tegangan sumber GGL yang nilainya tidak berubah baik sebelum salah satu lampu putusmaupun setelah salah satu lampu putus.

Hitung hambatan rangkaian setelah salah satu lampu putus :Jika salah satu lampu putus maka lampu lainnya yang masih menyala terangkai seri dengan resistor 2Ohm. Dengan demikian, hambatan rangkaian berubah menjadi :R = 6 Ohm + 2 Ohm = 8 Ohm

Hitung kuat arus listrik yang mengalir pada rangkaian setelah salah satu lampu putus :Kuat arus listrik yang mengalir pada rangkaian adalah :I = V / R = 10 Volt / 8 Ohm = 1,25 AmpereLampu dan resistor terangkai seri sehingga kuat arus listrik yang mengalir pada rangkaian = kuat aruslistrik yang mengalir pada lampu = kuat arus listrik yang mengalir pada resistor.

Hitung daya lampu :Setelah salah satu lampu putus, lampu lainnya menyala dengan daya sebesar :P = V I = (10 Volt)(1,25 Ampere) = 12,5 Watt

© 2015 Gurumuda.net | A. San Lohat, S.Pd. 69

Page 72: osn fisika smp

Pembahasan Soal OSN Fisika SMP Tingkat Kabupaten/Kota

Gerak elektron3. Soal Olimpiade Sains Nasional SMP Tingkat Kabupaten/Kota Tahun 2011 No.54Diketahui massa elektron 9,1 x 10-31 kg dan muatan elektron 1,6 x 10-19 C. Beda potensial yangdiperlukan untuk mempercepat sebuah elektron dari laju 1 x 105 m/s menjadi 3 x 105 m/s adalah....A. 28,4375 mVB. 56,875 mVC. 113,75 mVD. 227,5 mVPembahasanDiketahui :Massa elektron (m) = 9,1 x 10-31 kgMuatan elektron (q) = 1,6 x 10-19 CKelajuan awal (vo) = 1 x 105 m/s ---> vo

2 = 1 x 1010 m2/s2

Kelajuan akhir (vt) = 3 x 105 m/s ---> vt2 = 9 x 1010 m2/s2

Ditanya : Beda potensial (V) yang diperlukan untuk mempercepat sebuah elektronJawab :Perubahan energi potensial listrik = q V = (1,6 x 10-19 C) VPerubahan energi kinetik = EKt – EKo = 1/2 m (vt

2 – vo2) = 1/2 (9,1 x 10-31 kg)(9 x 1010 m2/s2 - 1 x 1010

m2/s2) = (4,55 x 10-31 kg)(8 x 1010 m2/s2) = 36,4 x 10-21 kg m2/s2

Energi bersifat kekal sehingga energi potensial listrik berubah menjadi energi kinetik :EPL yang hilang = EK yang muncul(1,6 x 10-19) V = 36,4 x 10-21

V = 36,4 x 10-21 / 1,6 x 10-19

V = 22,75 x 10-2 VoltV = 227,5 x 10-3 VoltV = 227,5 miliVoltJawaban yang benar adalah D.

4. Soal Olimpiade Sains Nasional SMP Tingkat Kabupaten/Kota Tahun 2011 No.56Di sebuah laboratorium seberkas elektron ditembakkan ke arah Utara. Dari atas diamati bahwa berkasitu berbelok ke kiri. Kemudian seberkas elektron ditembakkan ke arah Selatan. Dari atas diamati bahwaberkas itu berbelok ke kiri. Ini berarti di tempat itu terdapat....A. medan listrik yang arahnya ke BaratB. medan listrik yang arahnya ke TimurC. medan magnetik yang arahnya ke atasD. medan magnetik yang arahnya ke bawahPembahasanElektron bermuatan listrik sehingga dapat digerakkan oleh gaya listrik maupun gaya magnet ketikaelektron berada dalam medan listrik atau medan magnet. Perbedaannya adalah gaya listrik dapatmenggerakkan elektron baik ketika elektron diam maupun ketika elektron bergerak, sedangkan gayamagnet hanya bisa menggerakan elektron ketika elektron sedang bergerak. Perbedaan lainnya adalah gaya listrik searah dengan medan listrik, sedangkan gaya magnet tegak lurusdengan medan magnet. Jadi jika elektron digerakkan oleh gaya listrik maka arah gerak elektron samadengan arah medan listrik, sebaliknya bila elektron digerakkan oleh gaya magnet maka arah gerakelektron sama dengan arah gaya magnet yang tegak lurus dengan arah medan magnet.

Berdasarkan soal di atas tampak bahwa ketika elektron ditembakkan ke utara, elektron dibelokkan kekiri (ke arah barat) dan ketika elektron ditembakkan ke selatan, elektron dibelokkan ke kiri (ke arah

© 2015 Gurumuda.net | A. San Lohat, S.Pd. 70

Page 73: osn fisika smp

Pembahasan Soal OSN Fisika SMP Tingkat Kabupaten/Kota

timur). Jika elektron dibelokkan oleh gaya listrik maka arah gaya listrik yang sama dengan arah medanlistrik, selalu berubah-ubah. Arah medan listrik seharusnya konstan. Jadi elektron tidak dibelokkan olehgaya listrik. Dapat disimpulkan bahwa yang membelokkan elektron adalah gaya magnet. Untuk menentukan arahgaya magnet (arah elektron dibelokkan), arah medan magnet dan arah elektron ditembakkan, gunakanaturan tangan kanan.Terapkan aturan tangan kanan seperti penjelasan berikut. Rentangkan tangan kanan, tegakkan ibu jaridan rapatkan keempat jari lainnya. Posisikan agar ibu jari menujuk arah utara (arah gerak elektron) dankeempat jari menunjuk ke arah barat (arah gaya magnet = arah elektron dibelokkan), maka punggungtangan kanan menunjuk ke bawah (arah medan magnet). Sebaliknya posisikan agar ibu jari menunjukke arah selatan (arah gerak elektron) dan keempat jari menunjuk ke arah timur (arah gaya magnet =arah elektron dibelokkan), maka punggung tangan kanan menunjuk ke bawah (arah medan magnet).Dapat disimpulkan bahwa elektron berada di dalam medan magnet yang arahnya ke bawah. Jawaban yang benar adalah D.

© 2015 Gurumuda.net | A. San Lohat, S.Pd. 71

Page 74: osn fisika smp

Pembahasan Soal OSN Fisika SMP Tingkat Kabupaten/Kota

Hukum Ampere, GGL induksi, Transformator

Hukum Ampere1. Soal Olimpiade Sains Nasional SMP Tingkat Kabupaten/Kota Tahun 2010 No.55Menurut H.C. Oersted, arus listrik menimbulkan medan magnet. Dengan mengembangkan pendapatitu, maka kumparan berarus listrik juga menimbulkan medan magnet. Kumparan yang jika beraruslistrik menghasilkan medan magnet yang serupa medan magnet yang ditimbulkan oleh sebuah magnetbatang adalah...A. toroidaB. selenoidaC. kawat melingkarD. kumparan berbentuk persegi panjangPembahasan

Magnet batang mempunyai dua kutub magnet yakni kutub utara dan selatan. Medanmagnet yang ditimbulkan oleh magnet batang jika digambarkan berupa garis-garismedan magnet, garis-garis tersebut keluar dari kutub utara dan masuk ke kutub selatan.Medan magnet yang ditimbulkan oleh magnet batang serupa dengan medan magnet yang

ditimbulkan oleh selenoida, seperti pada gambar di samping. Selenoida adalah kumparan kawat panjang yang mempunyai banyak lilitan. Gambardi samping menunjukkan contoh pembuatan magnet dengan cara dililiti kumparankawat berarus listrik. arah kutub magnet dapat diketahui menggunakan kaidahtangan kanan. Rentangkan tangan kanan, posisikan agar telapak tangan kanan menghadap ke wajah. Tegakkan ibu jaridan tekuk keempat jari lainnya ke depan menuju wajah. Arah tekukan keempat jari sama dengan arahaliran arus listrik pada kumparan, sedangkan ibu jari menunjuk ke arah kutub utara medan magnet. Ibujari menujuk ke arah N sehingga N merupakan kutub utara magnet dan M merupakan kutub selatanmagnet. Jawaban yang benar adalah B.

Toroida2. Soal Olimpiade Sains Nasional SMP Tingkat Kabupaten/Kota Tahun 2010 No.59Medan magnet yang ditimbulkan oleh toroida yang dialiri arus listrik mempunyai arah....A. tegak lurus bidang toroidaB. melingkarC. bolak balikD. bergantung waktuPembahasanToroida adalah selenoida yang berbentuk lingkaran sehingga menggunakan kaidah tangan kanan,medan magnet yang ditimbulkan toroida mempunyai arah melingkar.Jawaban yang benar adalah B.

GGL Induksi3. Soal Olimpiade Sains Nasional SMP Tingkat Kabupaten/Kota Tahun 2010 No.58Sebuah magnet yang digerakkan masuk ke dalam sebuah kumparan, maka antara ujung-ujungkumparan akan terjadi gaya gerak listrik induksi, peristiwa ini disebabkan karena....A. terjadi perubahan arah medan magnet yang memotong kumparanB. terjadi perubahan kutub-kutub magnet yang memotong kumparanC. terjadi perubahan jumlah garis gaya magnet yang memotong kumparan

© 2015 Gurumuda.net | A. San Lohat, S.Pd. 72

Page 75: osn fisika smp

Pembahasan Soal OSN Fisika SMP Tingkat Kabupaten/Kota

D. terjadi pengurangan jumlah garis gaya magnet yang memotong kumparanPembahasan

Gaya gerak listrik induksi dihasilkan oleh medan magnet yang berubah. Besarnya GGL induksitidak bergantung pada laju perubahan medan magnet tetapi bergantung pada laju perubahanfluks magnetik. Fluks magnetik adalah jumlah garis medan magnet yang melewati kumparan.Secara matematis rumus fluks magnetik adalah ΦB = B A cos θ. Jika garis medan magnet tegaklurus luas permukaan kumparan maka sudut yang terbentuk antara garis medan magnet dengangaris normal adalah 0o, di mana cosinus 0o = 1. Apabila garis medan magnet sejajar dengan

luas permukaan kumparan maka sudut yang terbentuk adalah 90o, di mana cosinus 90o = 0. Jadi fluksmagnetik bernilai minimum ketika garis medan magnet sejajar dengan luas permukaan kumparan danbernilai maksimum ketika garis medan magnet tegak lurus dengan luas permukaan kumparan.Besar ggl induksi dihitung menggunakan rumus hukum induksi Faraday :

ξ=− NΔΦB

ΔtKeterangan :

ξ = ggl induksiΔΦB

Δt= laju perubahan fluks magnetik

N = jumlah lilitan kawatTanda negatif menunjukkan bahwa gaya gerak listrik induksi membangkitkan arus listrik yang arahnyaberlawanan dengan asal perubahan fluks. Pernyataan ini disebut hukum Lenz.Jawaban yang benar adalah C.

4. Soal Olimpiade Sains Nasional SMP Tingkat Kabupaten/Kota Tahun 2011 No.58GGL induksi yang timbul pada suatu kumparan besarnya bertambah secara linear terhadap waktu. Jikaluas permukaan kumparan tetap dan medan magnet berarah tegak lurus permukaan kumparan, makagrafik medan magnet yang mungkin untuk kondisi ini adalah....A C

B D

PembahasanBesar ggl induksi dihitung menggunakan rumus hukum induksi Faraday :

ξ=− NΔΦB

ΔtJika medan listrik tegak lurus permukaan kumparan maka :

ξ=− NΔΦB

ΔtKeterangan :

ξ = ggl induksi,ΔΦB

Δt= laju perubahan fluks magnetik

Jika medan magnet tegak lurus permukaan kumparan maka rumus fluks magnet adalah :

© 2015 Gurumuda.net | A. San Lohat, S.Pd. 73

Page 76: osn fisika smp

Pembahasan Soal OSN Fisika SMP Tingkat Kabupaten/Kota

ΔΦB=B A cosθ=B A cos 0o=B A (1 )=BA

B = medan magnet, A = luas permukaan kumparan.Luas permukaan kumparan bernilai tetap dan medan magnet berarah tegak lurus permukaan kumparansehingga bernilai tetap.Jadi berdasarkan rumus di atas, dapat disimpulkan bahwa yang mempengaruhi nilai ggl induksi adalahlaju perubahan fluks, bila laju perubahan fluks semakin besar maka GGL induksi semakin besar. Jawaban yang benar adalah B.

Transformator5. Soal Olimpiade Sains Nasional SMP Tingkat Kabupaten/Kota Tahun 2010 No.54Perhatikan pernyataan berikut!1. Jumlah lilitan primer lebih sedikit daripada jumlah lilitan sekunder2. Tegangan primer lebih besar daripada tegangan sekunder3. Arus primer lebih besar daripada arus sekunder4. Daya primer lebih kecil daripada daya sekunderPernyataan yang sesuai dengan ciri-ciri trafo step up adalah....A. pernyataan 1 dan 2B. pernyataan 1 dan 3C. pernyataan 2 dan 3D. pernyataan 2 dan 4PembahasanTransformator atau trafo adalah alat untuk mengubah tegangan listrik arus bolak balik (AC). Trafoterdiri dari dua jenis yakni trafo step up dan trafo step down. Trafo step up berfungsi menaikkantegangan dan trafo step down berfungsi menurunkan tegangan listrik.

Trafo step up berfungsi menaikan tegangansehingga ciri-cirinya adalah :1. Tegangan primer lebih kecil, tegangan sekunderlebih besar2. Arus listrik primer lebih besar, arus listriksekunder lebih kecil3. Jumlah lilitan primer lebih sedikit, jumlahlilitan sekunder lebih banyak

Ciri-ciri trafo step down berfungsi menurunkantegangan sehingga ciri-cirinya adalah :1. Tegangan primer lebih besar, tegangan sekunderlebih kecil2. Arus listrik primer lebih kecil, arus listriksekunder lebih besar2. Jumlah lilitan primer lebih banyak, jumlahlilitan sekunder lebih sedikit Daya listrik adalah energi listrik yang digunakan selama selang waktu tertentu. Energi bersifat kekalkarenanya daya juga bersifat kekal. Jadi energi listrik pada kumparan primer sama dengan energi listrikpada kumparan sekunder, demikian juga dengan daya listrik. Rumus daya listrik :P = V IKeterangan : P = daya listrik, V = tegangan listrik, I = kuat arus listrik

© 2015 Gurumuda.net | A. San Lohat, S.Pd. 74

Page 77: osn fisika smp

Pembahasan Soal OSN Fisika SMP Tingkat Kabupaten/Kota

Daya konstan karenanya berdasarkan rumus ini disimpulkan bahwa jika tegangan listrik besar makakuat arus listrik kecil agar hasil kali antara tegangan listrik dan kuat arus listrik bernilai konstan.Jawaban yang benar adalah B.

© 2015 Gurumuda.net | A. San Lohat, S.Pd. 75